Brilliant Integration Contest - Season 3

Hi Brilliant! Here is the most awaited Integration contest! See Season 1 and Season 2.

The aims of the Integration contest are to improve skills in the computation of integrals, to learn from each other as much as possible, and of course to have fun. Anyone here may participate in this contest.

The rules are as follows:

  • I will start by posting the first problem. If there is a user solves it, then they must post a new one.

  • You may only post a solution of the problem below the thread of problem and post your proposed problem in a new thread. Put them separately.

  • Only make substantial comment that will contribute to the discussion.

  • Make sure you know how to solve your own problem before posting it in case there is no one can answer it within 48 hours, then you must post the solution and you have a right to post another problem.

  • If the one who solves the last problem does not post his/her own problem after solving it within a day, then the one who has a right to post a problem is the last solver before him/her.

  • The scope of questions is only computation of integrals either definite or indefinite integrals.

  • You are NOT allowed to post a multiple integrals problem.

  • It is NOT compulsory to post original problems. But make sure it has not been posted on brilliant.

  • Do not copy questions from last year's contest. If anyone found to do so he/she will be banned from taking further part in this contest

  • Direct problems on contour integration are not allowed. However, solutions which use contour integration are allowed.

  • Also don't forget to upvote good problems and solutions!

Format your post is as follows:

1
2
3
4
5
6
7
**SOLUTION OF PROBLEM xxx (number of problem) :**

**[Post your solution here]**

**PROBLEM xxx (number of problem) :**

**[Post your problem here]**

The comments will be easiest to follow if you sort by "Newest":

Continue here for part 2 of the contest.

#Calculus

Note by Aditya Kumar
4 years, 6 months ago

No vote yet
1 vote

  Easy Math Editor

This discussion board is a place to discuss our Daily Challenges and the math and science related to those challenges. Explanations are more than just a solution — they should explain the steps and thinking strategies that you used to obtain the solution. Comments should further the discussion of math and science.

When posting on Brilliant:

  • Use the emojis to react to an explanation, whether you're congratulating a job well done , or just really confused .
  • Ask specific questions about the challenge or the steps in somebody's explanation. Well-posed questions can add a lot to the discussion, but posting "I don't understand!" doesn't help anyone.
  • Try to contribute something new to the discussion, whether it is an extension, generalization or other idea related to the challenge.
  • Stay on topic — we're all here to learn more about math and science, not to hear about your favorite get-rich-quick scheme or current world events.

MarkdownAppears as
*italics* or _italics_ italics
**bold** or __bold__ bold

- bulleted
- list

  • bulleted
  • list

1. numbered
2. list

  1. numbered
  2. list
Note: you must add a full line of space before and after lists for them to show up correctly
paragraph 1

paragraph 2

paragraph 1

paragraph 2

[example link](https://brilliant.org)example link
> This is a quote
This is a quote
    # I indented these lines
    # 4 spaces, and now they show
    # up as a code block.

    print "hello world"
# I indented these lines
# 4 spaces, and now they show
# up as a code block.

print "hello world"
MathAppears as
Remember to wrap math in \( ... \) or \[ ... \] to ensure proper formatting.
2 \times 3 2×3 2 \times 3
2^{34} 234 2^{34}
a_{i-1} ai1 a_{i-1}
\frac{2}{3} 23 \frac{2}{3}
\sqrt{2} 2 \sqrt{2}
\sum_{i=1}^3 i=13 \sum_{i=1}^3
\sin \theta sinθ \sin \theta
\boxed{123} 123 \boxed{123}

Comments

Problem 6:

Prove that 0lnx(x+a)2+b2dx  =  1btan1balna2+b2 \int_0^\infty \frac{\ln x}{(x+a)^2 + b^2}\,dx \; = \; \tfrac{1}{b}\,\tan^{-1}\tfrac{b}{a}\,\ln\sqrt{a^2+b^2} for all a,b>0a,b > 0.

This problem has been solved by Aditya Sharma.

Mark Hennings - 4 years, 6 months ago

Log in to reply

Solution to problem 6 :

Let us denote the integral by II .

I=0lnx(x+a)2+b2dx=0lnx(x+a+ib)(x+aib)\displaystyle I = \int_0^\infty \frac{\ln x}{(x+a)^2+b^2}dx = \int_0^\infty \frac{\ln x}{(x+a+ib)(x+a-ib)}

With the substitution xy=a2+b2xy=a^2+b^2 we have,

I=ln(a2+b2)0dyy2+2ay+a2+b20lny(y+a)2+b2dyI\displaystyle I=\ln(a^2+b^2)\int_0^\infty \frac{dy}{y^2+2ay+a^2+b^2} - \underbrace{\int_0^\infty \frac{\ln y}{(y+a)^2+b^2}dy}_I

I=ln(a2+b2)0dx(x+a)2+b2=1btan1balna2+b2\displaystyle I=\ln(\sqrt{a^2+b^2})\int_0^\infty \frac{dx}{(x+a)^2+b^2} = \frac{1}{b}\tan^{-1}\frac{b}{a}\ln\sqrt{a^2+b^2}.

Aditya Narayan Sharma - 4 years, 6 months ago

As an alternative to Aditya's proof...

Consider the cut plane C\[0,)\mathbb{C} \backslash [0,\infty), so that 0<Argz<2π0 < \mathrm{Arg}\,z < 2\pi for all zz, and consider the integral of f(z)  =  (logz)2(z+a)2+b2 f(z) \; = \; \frac{(\log z)^2}{(z+a)^2 + b^2} around the keyhole contour γ1+γ2γ3γ4\gamma_1 + \gamma_2 - \gamma_3 - \gamma_4, where

  • γ1\gamma_1 is the line segment z=xe0iz \,=\, x e^{0i}, for ε<x<R\varepsilon < x < R, just above the cut,
  • γ2\gamma_2 is the circular contour z=Reiθz \,=\, Re^{i\theta} for 0<θ<2π0 < \theta < 2\pi,
  • γ3\gamma_3 the the line segment z=xe2πiz \,=\, x e^{2\pi i}, for ε<x<R\varepsilon < x < R, just below the cut,
  • γ4\gamma_4 is the circular contour z=εeiθz \,=\, \varepsilon e^{i\theta} for 0<θ<2π0 < \theta < 2\pi.

where we assume that 0<ε<a2+b2<R0 < \varepsilon < \sqrt{a^2+b^2} < R. Now γ1f(z)dz=εR(lnx)2(x+a)2+b2dxγ3f(z)dz=εR(lnx+2πi)2(x+a)2+b2dx(γ1γ3)f(z)dz=εR4π24πilnx(x+a)2+b2dx\begin{aligned} \int_{\gamma_1} f(z)\,dz & = \int_\varepsilon^R \frac{(\ln x)^2}{(x+a)^2 + b^2}\,dx \\ \int_{\gamma_3} f(z)\,dz & = \int_\varepsilon^R \frac{(\ln x + 2\pi i)^2}{(x+a)^2 + b^2}\,dx \\ \left(\int_{\gamma_1} - \int_{\gamma_3}\right)\,f(z)\,dz & = \int_{\varepsilon}^R \frac{4\pi^2 - 4\pi i \ln x}{(x+a)^2 + b^2}\,dx \end{aligned} We also note that γ2f(z)dz  =  O(ln2RR)γ4f(z)dz  =  O(εln2ε) \int_{\gamma_2} f(z)\,dz \; = \; O\big(\tfrac{\ln^2R}{R}\big) \hspace{2cm} \int_{\gamma_4} f(z)\,dz \; = \; O\big(\varepsilon \ln^2\varepsilon\big) as RR \to \infty and ε0\varepsilon \to 0. Letting RR \to \infty and ε0\varepsilon \to 0, we deduce that 04π24πilnx(x+a)2+b2dx  =  2πi[Resz=a+ibf(z)+Resz=aibf(z)] \int_0^\infty \frac{4\pi^2 - 4\pi i \ln x}{(x+a)^2 + b^2}\,dx \; = \; 2\pi i\Big[ \mathrm{Res}_{z=-a+ib} f(z) \,+\, \mathrm{Res}_{z=-a-ib} f(z)\Big] Now f(z)f(z) has simple poles at each of a±ib-a \pm ib, and Resz=a±ibf(z)=(lna2+b2+iπitan1ba)2±2ibResz=a+ibf(z)+Resz=aibf(z)=4i(lna2+b2+iπ)tan1ba2ib=2btan1ba(lna2+b2+iπ)\begin{aligned} \mathrm{Res}_{z = -a\pm ib} f(z) & = \frac{\Big(\ln\sqrt{a^2+b^2} + i\pi \mp i\tan^{-1}\frac{b}{a}\Big)^2}{\pm 2ib} \\ \mathrm{Res}_{z = -a+ib} f(z) + \mathrm{Res}_{z = -a-ib} f(z) & = \frac{-4i(\ln\sqrt{a^2+b^2}+i\pi)\tan^{-1}\frac{b}{a}}{2ib} \\ & = - \frac{2}{b}\tan^{-1}\frac{b}{a}(\ln\sqrt{a^2+b^2} + i\pi) \end{aligned} and hence 04π24πilnx(x+a)2+b2dx  =  4πbtan1ba(πilna2+b2) \int_0^\infty \frac{4\pi^2 - 4\pi i \ln x}{(x+a)^2 + b^2}\,dx \; = \; \frac{4\pi}{b}\tan^{-1}\frac{b}{a}\big(\pi - i\ln\sqrt{a^2+b^2}\big) Taking imaginary parts of this equation gives the result.

Mark Hennings - 4 years, 6 months ago

Problem 16:

Evaluate the integral 01xa1(1x)b1(x+p)a+bdx \int_0^1 \frac{x^{a-1}(1-x)^{b -1}}{(x+p)^{a+b}}\,dx for all a,b,p>0a,b,p > 0.

This problem has been solved by Aditya Sharma.

Mark Hennings - 4 years, 5 months ago

Log in to reply

If we use the substitution, x=1u+1\displaystyle x=\frac{1}{u+1} then

I=0ub1(up+p+1)a+bdu\displaystyle I =\int_0^\infty \frac{u^{b-1}}{(up+p+1)^{a+b}}\,du

Again using the substitution, u=y(p+1p)\displaystyle u=y\left(\frac{p+1}{p}\right) the integral transforms to,

I=1pb(1+p)a0yb1(1+y)a+bdy=(β(a,b)pb(1+p)a)\displaystyle I =\frac{1}{p^b(1+p)^a}\int_0^\infty \frac{y^{b-1}}{(1+y)^{a+b}}\,dy = \left(\frac{\beta(a,b)}{p^b(1+p)^a}\right)

Aditya Narayan Sharma - 4 years, 5 months ago

Log in to reply

The substitution z=(p+1)xp+x z = \frac{(p+1)x}{p+x} Is even more direct. Not a hard problem, but a very elegant result.

Mark Hennings - 4 years, 5 months ago

Log in to reply

@Mark Hennings Yes, Although this substitution emerges by combining the chain of substitutions used, or it's tough to put it directly without knowing why it is the reqd. substitution.

Aditya Narayan Sharma - 4 years, 5 months ago

Problem 14 : Prove That

0lnx[ln(x+12)1x+1ψ(x+12)]dx=ln222+ln2lnπ1 \int_{0}^{\infty} \ln x\left[\ln \left( \dfrac{x+1}{2} \right) - \dfrac{1}{x+1} - \psi \left( \dfrac{x+1}{2} \right) \right] \mathrm{d}x = \dfrac{\ln^2 2}{2}+\ln2\cdot\ln\pi-1

Notation : ψ(x) \psi(x) denotes the Digamma Function.

This problem has been solved by Mark Hennings.

Ishan Singh - 4 years, 5 months ago

Log in to reply

This is a really nice use of previous results of this round of the contest!

Starting with the identity ψ(x)  =  lnx12x20t(t2+x2)(e2πt1)dt \psi(x) \; = \; \ln x - \frac{1}{2x} - 2\int_0^\infty \frac{t}{(t^2+x^2)(e^{2\pi t}-1)}\,dt we deduce that ln(x+12)+1x+1ψ(x+12)=20t(t2+14(x+1)2)(e2πt1)dt=80t(4t2+(x+1)2)(e2πt1)dt=20t(t2+(x+1)2)(eπt1)dt\begin{aligned} \ln\big(\tfrac{x+1}{2}\big) + \tfrac{1}{x+1} - \psi\big(\tfrac{x+1}{2}\big) & = 2\int_0^\infty \frac{t}{\big(t^2 + \frac14(x+1)^2\big)(e^{2\pi t} - 1)}\,dt \\ & = 8\int_0^\infty \frac{t}{(4t^2 + (x+1)^2)(e^{2\pi t} - 1)}\,dt \\ & = 2\int_0^\infty \frac{t}{(t^2 + (x+1)^2)(e^{\pi t}-1)}\,dt \end{aligned} and hence 0lnx(ln(x+12)+1x+1ψ(x+12))dx=20(0lnx(x+1)2+t2dx)teπt1dt=20(1ttan1tlnt2+1)teπt1dt  =  0tan1tln(t2+1)eπt1dt=12ln22+ln2lnπ1\begin{aligned} \int_0^\infty \ln x & \left(\ln\big(\tfrac{x+1}{2}\big) + \tfrac{1}{x+1} - \psi\big(\tfrac{x+1}{2}\big)\right)\,dx \\ & = 2\int_0^\infty \left(\int_0^\infty \frac{\ln x}{(x+1)^2 + t^2}\,dx\right)\,\frac{t}{e^{\pi t}-1}\,dt \\ & = 2\int_0^\infty \left(\frac{1}{t} \tan^{-1}t \ln\sqrt{t^2 + 1}\right)\,\frac{t}{e^{\pi t}-1}\,dt \; = \; \int_0^\infty \frac{\tan^{-1}t \ln(t^2+1)}{e^{\pi t} - 1}\,dt \\ & = \tfrac12\ln^22 + \ln 2 \ln\pi - 1 \end{aligned} using the results of Problems 55 and 66.

Ishan, please set another one!

Mark Hennings - 4 years, 5 months ago

Log in to reply

Yes indeed! This integral arose from my attempt to find an alternate solution of Problem 5 :) The first identity can be proved (using real analysis) by differentiating the identity 0log(1e2aπx)1+x2dx=π[12log(2aπ)+a(loga1)log(Γ(a+1))]\int_{0}^{\infty} \dfrac{\log(1-e^{-2a\pi x})}{1+x^2} \mathrm{d}x = \pi \left[\dfrac{1}{2} \log (2a\pi ) + a(\log a - 1) - \log(\Gamma(a+1)) \right] wrt aa. I have proved it here.

Ishan Singh - 4 years, 5 months ago

I have posted a question on Math Stack Exchange asking for alternate methods to evaluate the above integral (independently of Problems 55 and 66) here. If we can evaluate it using other methods, it'll also give us an alternate solution for Problem 55.

Ishan Singh - 4 years, 5 months ago

Problem 1:

Show that: 01cosh(alog(x))log(1+x)xdx=12a(πcsc(πa)1a),   a<1\int_{0}^{1}\frac{\cosh(a\log(x))\log(1+x)}{x}dx=\frac{1}{2a}(\pi \csc(\pi a)-\frac{1}{a}), \;\ a<1

This problem has been solved by Aditya Sharma.

Aditya Kumar - 4 years, 6 months ago

Log in to reply

If we denote the integral by II & using 2cosh(alnx)=xa+xa2cosh(a\ln x)=x^a+x^{-a} & ln(1+x)=n1(1)n1nxn\displaystyle \ln(1+x)=\sum_{n\ge 1}\frac{(-1)^{n-1}}{n}x^n the integral can be transformed into,

I=n1(1)n1n01(xn+a1+xna1)=n1(1)n1n(1na+1n+a)=12an1(1)n1(1na1n+a)\displaystyle I = \sum_{n\ge 1} \frac{(-1)^{n-1}}{n} \int_0^1 (x^{n+a-1}+x^{n-a-1})=\sum_{n\ge 1} \frac{(-1)^{n-1}}{n}(\frac{1}{n-a}+\frac{1}{n+a})=\frac{1}{2a}\sum_{n\ge 1} (-1)^{n-1} (\frac{1}{n-a}-\frac{1}{n+a})

For removing the alternating factor (1)n1(-1)^{n-1} this can be written as ,

2aI=n1(1na1n+a)2n1(12na12n+a)\displaystyle 2a I = \sum_{n\ge 1} (\frac{1}{n-a}-\frac{1}{n+a})-2\sum_{n\ge 1} (\frac{1}{2n-a}-\frac{1}{2n+a})

which simplifies to ,

2aI=n1(1na1n+a1na2+1n+a2)\displaystyle 2a I = \sum_{n\ge 1} (\frac{1}{n-a}-\frac{1}{n+a}-\frac{1}{n-\frac{a}{2}}+\frac{1}{n+\frac{a}{2}})

Using Ha=n1(1n1n+a)\displaystyle H_a = \sum_{n\ge 1}(\frac{1}{n}-\frac{1}{n+a}) we have,

I=12a(HaHaHa2+Ha2)\displaystyle I=\frac{1}{2a}(H_a-H_{-a}-H_{\frac{a}{2}}+H_{-\frac{a}{2}})

Again using reflection formula for harmonic numbers as a<1a<1

i.e. H1aHa=πcot(πa)1a+11a\displaystyle H_{1-a}-H_a = \pi \cot(\pi a)-\frac{1}{a}+\frac{1}{1-a} it simplifies to,

I=12a(πcot(πa)+πcot(πa2)1a)=12a(πcsc(πa)1a)\displaystyle I = \frac{1}{2a} (-\pi\cot(\pi a)+\pi\cot(\pi\frac{a}{2})-\frac{1}{a}) = \frac{1}{2a}(\pi\csc(\pi a)-\frac{1}{a})

Aditya Narayan Sharma - 4 years, 6 months ago

Problem 5:

Prove that:

0ln(x2+1)arctanxeπx1dx=ln222+ln2lnπ1.{\large\int}_0^\infty\frac{\ln\left(x^2+1\right)\,\arctan x}{e^{\pi x}-1}dx=\frac{\ln^22}2+\ln2\cdot\ln\pi-1.

This problem has been solved by Mark Hennings.

Aditya Kumar - 4 years, 6 months ago

Log in to reply

Solution of Problem 5

Starting with the Hermite formula for the generalized Riemann zeta function: ζ(s,a)  =  12as+a1s(s1)1+20(a2+y2)12ssin(stan1ya)e2πy1dy \zeta(s,a) \; = \; \tfrac12a^{-s} + a^{1-s}(s-1)^{-1} + 2\int_0^\infty \frac{(a^2+y^2)^{-\frac12s} \, \sin\big(s \tan^{-1}\frac{y}{a}\big)}{e^{2\pi y} - 1}\,dy we have ζ(s,12)=2s1(1+1s1)+20(14+y2)12ssin(stan12y)e2πy1dy=2s1ss1+2s+10(1+4y2)12ssin(tan12y)e2πy1dy=2s1ss1+2s0(1+t2)12ssin(stan1t)eπt1dt \begin{aligned} \zeta(s,\tfrac12) & = 2^{s-1}\left(1 + \tfrac{1}{s-1}\right) + 2\int_0^\infty \frac{(\frac14 + y^2)^{-\frac12s}\,\sin\big(s \tan^{-1}2y\big)}{e^{2\pi y}-1}\,dy \\ & = 2^{s-1}\frac{s}{s-1} + 2^{s+1}\int_0^\infty \frac{(1 + 4y^2)^{-\frac12s} \, \sin\big(\tan^{-1}2y\big)}{e^{2\pi y} - 1}\,dy \\ & = 2^{s-1}\frac{s}{s-1} + 2^s \int_0^\infty \frac{(1+t^2)^{-\frac12s}\,\sin\big(s \tan^{-1}t\big)}{e^{\pi t}-1}\,dt \end{aligned} so that F(s)  =  0(1+t2)12ssin(stan1t)eπt1dt  =  2sζ(s,12)s2(s1) F(s) \; = \; \int_0^\infty \frac{(1+t^2)^{-\frac12s}\,\sin(s \tan^{-1}t\big)}{e^{\pi t}-1}\,dt \; = \; 2^{-s}\zeta(s,\tfrac12) - \frac{s}{2(s-1)} Now since F(s)=2s20(1+t2)12ssin(stan1t)eπt1dt=0(1+t2)12seπt1{14(ln(1+t2))2sin(stan1t)ln(1+t2)tan1tcos(stan1t)(tan1t)2sin(stan1t)}dt \begin{aligned} F''(s) & = \frac{\partial^2}{\partial s^2}\int_0^\infty \frac{(1 + t^2)^{-\frac12s} \, \sin(s \tan^{-1}t\big)}{e^{\pi t}-1}\,dt \\ & = \int_0^\infty \frac{(1+t^2)^{-\frac12s}}{e^{\pi t} - 1}\left\{ \begin{array}{l} \tfrac14\big(\ln(1+t^2)\big)^2 \sin(s\tan^{-1}t) - \ln(1 + t^2) \tan^{-1}t \cos\big(s \tan^{-1}t\big) \\ - \big(\tan^{-1}t\big)^2 \sin\big(s \tan^{-1}t\big)\end{array}\right\}\,dt \end{aligned} we deduce that F(0)  =  0ln(1+t2)tan1teπt1dt F''(0) \; = \; -\int_0^\infty \frac{\ln(1+t^2) \tan^{-1}t}{e^{\pi t}-1}\,dt and hence that the desired integral is 0ln(1+t2)tan1teπt1dt  =  F(0)  =  2s2[s2(s1)2sζ(s,12)]s=0 \int_0^\infty \frac{\ln(1+t^2) \tan^{-1}t}{e^{\pi t}-1}\,dt \; = \; -F''(0) \; = \; \frac{\partial^2}{\partial s^2}\Big[\tfrac{s}{2(s-1)} - 2^{-s}\zeta(s,\tfrac12)\Big] \Big|_{s=0} which simplifies to give 0ln(1+t2)tan1teπt1dt  =  112ln22+ln2ln2π  =  1+12ln22+ln2lnπ \int_0^\infty \frac{\ln(1+t^2) \tan^{-1}t}{e^{\pi t}-1}\,dt \; = \; -1 - \tfrac12\ln^22 + \ln2 \cdot \ln2\pi \; = \; -1 + \tfrac12\ln^22 + \ln2\ln\pi

Mark Hennings - 4 years, 6 months ago

Problem 8:

Show that 012πln2(asinθ)dθ  =  124π3+16π[ln2(2a)2ln(2a)ln(a2)] \int_0^{\frac12\pi} \ln^2\big(a\sin\theta\big)\,d\theta \; = \; \tfrac{1}{24}\pi^3 + \tfrac{1}{6}\pi\Big[\ln^2\big(\tfrac{2}{a}\big) - 2\ln\big(\tfrac{2}{a}\big)\ln\big(\tfrac{a}{2}\big)\Big] for any a>0a > 0.

This problem has been solved by Jasper Braun.

Mark Hennings - 4 years, 6 months ago

Log in to reply

Solution to Problem 8: I=0π2ln2a+2lnalnsinx+ln2sinxdx=πln2a2lnaπln2+0π2ln2sinxdx\displaystyle I=\int_0^\frac{\pi}{2}\ln^2{a}+2\ln{a}\ln{\sin{x}}+\ln^2{\sin{x}}dx=\frac{\pi\ln^2{a}}{2}-\ln{a}\pi\ln{2}+\int_0^\frac{\pi}{2}\ln^2{\sin{x}}dx

0π2ln2sinxdx=01ln2x1x2dx(by u=sinx)\displaystyle\int_0^\frac{\pi}{2}\ln^2{\sin{x}}dx = \int_0^1\frac{\ln^2{x}}{\sqrt{1-x^2}}dx\quad\text{(by }u= \sin{x})

Now using B(m,n)=201x2m1(1x2)n1dx and thus 01ln2x1x2dx=d2dm2B(m,12)2412\displaystyle\text{Now using }B(m,n) = 2\int_0^1x^{2m-1}(1-x^2)^{n-1}dx \text{ and thus }\int_0^1\frac{\ln^2{x}}{\sqrt{1-x^2}}dx = \frac{d^2}{dm^2}\frac{B(m,\frac{1}{2})}{2*4}\big|_\frac{1}{2}

Using Γ(m)=Γ(m)ψ(m),Γ(m)=Γ(m)ψ(m)2+Γ(m)ψ(1)(m)\displaystyle\text{Using }\Gamma'(m) = \Gamma(m)\psi(m),\qquad\Gamma''(m) = \Gamma(m)\psi(m)^2+\Gamma(m)\psi^{(1)}(m)

π8d2dm2Γ(m)Γ(m+12)12=πln222+πζ(2)4\displaystyle\frac{\sqrt{\pi}}{8}\frac{d^2}{dm^2}\frac{\Gamma(m)}{\Gamma(m+\frac{1}{2})}\Big|_\frac{1}{2} = \frac{\pi\ln^2{2}}{2}+\frac{\pi\zeta(2)}{4}

I=π324+πln222+πln2a2πlnaln2\displaystyle I=\boxed{\frac{\pi^3}{24}+\frac{\pi\ln^2{2}}{2}+\frac{\pi\ln^2{a}}{2}-\pi\ln{a}\ln{2}}

First Last - 4 years, 6 months ago

Problem 13:

Evaluate 02πecosθcos(nθsinθ)dθ \int_0^{2\pi} e^{\cos\theta} \cos\big(n\theta - \sin\theta\big)\,d\theta for any integer nn.

This problem has been solved by Ishan Singh.

Mark Hennings - 4 years, 6 months ago

Log in to reply

It's simplest using contour integration...

Note that 02πecosθcos(sinθnθ)dθ  =  Re(02πecosθei(sinθnθ)dθ)  =  Re(02πeeiθinθdθ)\int_0^{2\pi} e^{\cos\theta}\,\cos(\sin\theta - n\theta)\,d\theta \; = \; \mathfrak{Re}\left(\int_0^{2\pi} e^{\cos\theta} e^{i(\sin\theta - n\theta)}\,d\theta\right) \; = \; \mathfrak{Re}\left(\int_0^{2\pi} e^{e^{i\theta} - in\theta}\,d\theta\right) With the substitution z=eiθz = e^{i\theta}, this becomes the contour integral Re(z=1ezzndziz)  =  Re(1iz=1ezzn+1dz)  =  2πRe(Resz=0ezzn+1)  =  {2πn!n00n<0 \mathfrak{Re}\left(\int_{|z|=1}e^z z^{-n} \frac{dz}{iz}\right) \; = \; \mathfrak{Re}\left(\frac{1}{i}\int_{|z|=1}\frac{e^z}{z^{n+1}}\,dz\right) \; = \; 2\pi\mathfrak{Re}\left(\mathrm{Res}_{z=0} \frac{e^z}{z^{n+1}}\right) \; = \; \left\{ \begin{array}{ll} \frac{2\pi}{n!} & n \ge 0 \\ 0 & n < 0 \end{array} \right.

Mark Hennings - 4 years, 5 months ago

I=02πecostcos(ntsint) dt=(02πeinte(costisint) dt) \text{I} = \int_{0}^{2 \pi} e^{\cos t} \cos (nt - \sin t) \ \mathrm{d}t = \Re{ \left(\int_{0}^{2 \pi} e^{i n t} \cdot e^{(\cos t - i \sin t)} \ \mathrm{d}t \right)}

=(02πeinteeit dt) = \Re{ \left(\int_{0}^{2 \pi} e^{i n t} \cdot e^{\Large e^{ -i t}} \ \mathrm{d}t \right)}

=2πn!+(r0 ; rn(1r!02πeit(nr) dt)) = \dfrac{2 \pi}{n!} + \Re{ \left(\sum_{r \geq 0 \ ; \ r \neq n} \left( \dfrac{1}{r!} \int_{0}^{2 \pi} e^{i t (n-r)} \ \mathrm{d}t \right) \right)}

Since last integral is 00, we have,

I=2πn! \text{I} = \dfrac{2 \pi}{n!}

If n n is negative, we can write n=k ; kZ+n = -k \ ; \ k \in \mathbb{Z}^+, so that,

J=02πecostcos(kt+sint) dt \text{J} = \int_{0}^{2 \pi} e^{\cos t} \cos (kt + \sin t) \ \mathrm{d}t

By the above method, we can similarly show that J=0 \text{J} = 0 .

Therefore,

02πecostcos(ntsint) dt={2πn!n00n<0 \int_{0}^{2 \pi} e^{\cos t} \cos (nt - \sin t) \ \mathrm{d}t = \left\{ \begin{array}{ll} \frac{2\pi}{n!} & n \ge 0 \\ 0 & n < 0 \end{array} \right.

Ishan Singh - 4 years, 5 months ago

Problem 2:

Evaluate 01(x2+1)tan13xxdx\large \displaystyle \int_{0}^{1} \frac{(x^{2}+1)\tan^{-1}3x}{x} \, dx.

This problem has been solved by Aditya Sharma.

Rohith M.Athreya - 4 years, 6 months ago

Log in to reply

The integral can be written as : I=01xtan1(3x)dxI1+03tan1xxdxI2\displaystyle I=\underbrace{\int_0^1 x\tan^{-1} (3x) dx}_{I_1}+\underbrace{\int_0^3 \frac{\tan^{-1}x}{x}dx}_{I_2}

A simple integration by parts would yield I1=59tan1316\displaystyle I_1 = \frac{5}{9}\tan^{-1}3 - \frac{1}{6}

I2I_2 is the Inverse Tangent Integral and we represent it by dilogarithms.

Ti2(3)=12i(Li2(3i)Li2(3i))\displaystyle Ti_2(3)=\frac{1}{2i}(Li_2(3i)-Li_2(-3i))

So, I=59tan1316+12i(Li2(3i)Li2(3i))\displaystyle I = \frac{5}{9}\tan^{-1}3 - \frac{1}{6} +\frac{1}{2i}(Li_2(3i)-Li_2(-3i))

Aditya Narayan Sharma - 4 years, 6 months ago

Problem 03:

Show that 01lnxln2(1+x)xdx=π4244Li4(12)7ζ(3)ln22+π2ln226ln426\displaystyle \int_0^1 \frac{\ln x \ln^2(1+x)}{x}\, dx = \dfrac{\pi^4}{24}-4{\rm Li}_4\left(\frac{1}{2}\right)-\frac{7\zeta(3)\ln 2}{2}+\frac{\pi^2\ln^2 2}{6}-\frac{\ln^4 2}{6}.

This problem has been solved by Jasper Braun.

Aditya Narayan Sharma - 4 years, 6 months ago

Log in to reply

Solution to Problem 3:

Hn(k)=j=1Hn(k)xiinH(x)=ln(1x)1x\displaystyle H_n^{(k)} = \sum_{j=1}^{\infty}\frac{H_n^{(k)}x^i}{i^n}\quad H(x) = \frac{-\ln{(1-x)}}{1-x}

I=0ln2(1+x)lnxxdx=01ln(1+x)ln2x1+xdx=by parts\displaystyle I = \int_{0}^{\infty}\frac{\ln^2{(1+x)}\ln{x}}{x}dx = -\int_{0}^{1}\frac{\ln{(1+x)}\ln^2{x}}{1+x}dx=\quad\text{by parts}

12ln2(x1)lnxxdx=(by u=1+x)\displaystyle -\int_{1}^{2}\frac{\ln^2{(x-1)}\ln{x}}{x}dx=\quad\text{(by } u = 1+x)

121ln2(1x1)ln1xxdx=121lnxln2(1x)x2ln2xln(1t)x+ln3xxdx(by u=1/x)\displaystyle -\int_{\frac{1}{2}}^{1}\frac{\ln^2{(\frac{1}{x}-1)}\ln{\frac{1}{x}}}{x}dx=\int_{\frac{1}{2}}^{1}\frac{\ln{x}\ln^2{(1-x)}}{x}-\frac{2\ln^2{x}\ln{(1-t)}}{x}+\frac{\ln^3{x}}{x}dx\quad\text{(by }u = 1/x)

lnxln2(1x)x=ln2x2ln2(1x)+ln2xln(1x)1xdx\displaystyle\int\frac{\ln{x}\ln^2{(1-x)}}{x} = \frac{\ln^2{x}}{2}\ln^2{(1-x)}+\int\frac{\ln^2{x}\ln{(1-x)}}{1-x}dx

ln2xln(1x)1xdx=n=1Hnd2dn2xndx=n=1Hn(xn+1ln2xn+12xn+1lnx(1+n)2+2xn+1(1+n)3)=\displaystyle \int\frac{\ln^2{x}\ln{(1-x)}}{1-x}dx = -\sum_{n=1}^{\infty}H_n\frac{d^2}{dn^2}\int x^ndx = -\sum_{n=1}^{\infty}H_n\big(\frac{x^{n+1}\ln^2{x}}{n+1}-\frac{2x^{n+1}\ln{x}}{(1+n)^2}+\frac{2x^{n+1}}{(1+n)^3}\big)=

Now use Hn=Hn+11n+1H_n = H_{n+1} - \frac{1}{n+1} to obtain:

(H1(x)ln2xLi2(x)ln2x2H2(x)lnx+2Li3(x)lnx+2H3(x)2Li4(x))\displaystyle -\Big(H_1(x)\ln^2{x}-Li_2(x)\ln^2{x}-2H_2(x)\ln{x}+2Li_3(x)\ln{x}+2H_3(x)-2Li_4(x)\Big)

Next 2ln2xln(1x)xdx=2n=11nd2dn2xn1=\displaystyle\textbf{Next }-2\int\frac{\ln^2{x}\ln{(1-x)}}{x}dx = 2\sum_{n=1}^{\infty}\frac{1}{n}\frac{d^2}{dn^2}\int x^{n-1} =

2n=1xnln2xn22xnlnxn3+2xnn4=2(Li2(x)ln2x2Li3(x)lnx+2Li4(x))\displaystyle 2\sum_{n=1}^{\infty}\frac{x^n\ln^2{x}}{n^2}-\frac{2x^n\ln{x}}{n^3}+\frac{2x^n}{n^4} = 2\big(Li_2(x)\ln^2{x}-2Li_3(x)\ln{x}+2Li_4(x)\big)

I=ln4x4121+ln2x2ln2(1x)1212H3(1)+2Li4(1)+\displaystyle I = \frac{\ln^4{x}}{4}\Big|^1_{\frac{1}{2}} + \frac{\ln^2{x}}{2}\ln^2{(1-x)}\Big|^1_{\frac{1}{2}}-2H_3(1)+2Li_4(1)+

+H1(12)ln22Li2(12)ln22+2H2(12)ln22Li3(12)ln2+2H3(12)2Li4(12)+\displaystyle +H_1(\frac{1}{2})\ln^2{2}-Li_2(\frac{1}{2})\ln^2{2}+2H_2(\frac{1}{2})\ln{2}-2Li_3(\frac{1}{2})\ln{2}+2H_3(\frac{1}{2})-2Li_4(\frac{1}{2})+

4Li4(1)2Li2(12)ln224Li3(12)ln24Li4(12)=\displaystyle4Li_4(1)-2Li_2(\frac{1}{2})\ln^2{2}-4Li_3(\frac{1}{2})\ln{2}-4Li_4(\frac{1}{2}) =

3ln4242H3(1)+H1(12)ln22+2H2(12)ln2+2H3(12)6Li4(12)+6Li4(1)3Li2(12)ln226Li3(12)ln2\displaystyle\frac{-3ln^4{2}}{4}-2H_3(1)+H_1(\frac{1}{2})\ln^2{2}+2H_2(\frac{1}{2})\ln{2}+2H_3(\frac{1}{2})-6Li_4(\frac{1}{2})+6Li_4(1)-3Li_2(\frac{1}{2})\ln^2{2}-6Li_3(\frac{1}{2})\ln{2}

Now using this for the evaluation of Hn(x)H_n(x)

2H3(1)=π436,ln22H1(12)=ln422+ln22Li212,2ln2H212=2ln2ζ(3)ζ(2)ln22,2H3(12)=π4360+ln4212ζ(3)ln24+2Li4(12)\displaystyle -2H_3(1) = \frac{-\pi^4}{36},\ln^2{2}H_1(\frac{1}{2})=\frac{\ln^4{2}}{2}+\ln^2{2}Li_2{\frac{1}{2}},2\ln{2}H_2{\frac{1}{2}} = 2\ln{2}\zeta(3)-\zeta(2)\ln^2{2},2H_3(\frac{1}{2})= \frac{\pi^4}{360}+\frac{\ln^4{2}}{12}-\frac{\zeta(3)\ln{2}}{4}+2Li_4(\frac{1}{2})

Adding all this together and using formulas for Li3Li_3 and Li2Li_2(I have checked on a calculator) gets the desired answer. Anyone may post the next problem.

First Last - 4 years, 6 months ago

Problem 4:

Prove that 01Li32(x)x2dx=916ζ2(3)34ζ(2)ζ(3)14ζ2(2)+74ζ(4)3ζ(3)ln2+6ζ(3)6ζ(2)ln2+6ζ(2)12ln22\displaystyle \int_0^1 \frac{{\rm Li}_3^2(-x)}{x^2}\, dx = -\frac{9}{16}\zeta^2(3)-\frac{3}{4}\zeta(2)\zeta(3)-\frac{1}{4}\zeta^2(2)+\frac{7}{4}\zeta(4)-3\zeta(3)\ln 2+6\zeta(3)-6\zeta(2)\ln 2+6\zeta(2)-12\ln^2 2

This problem has been solved by Aditya Kumar.

Aditya Narayan Sharma - 4 years, 6 months ago

Log in to reply

Solution to Problem 4:

I=01Li32(x)x2dx\displaystyle I=\int^1_0\frac{Li_3^2(-x)}{x^2}{\rm d}x

I=Li32(x)x01+012Li2(x)Li3(x)x2dx=916ζ2(3)2Li2(x)Li3(x)x01+012Li22(x)x2dx012Li3(x)ln(1+x)x2dx I=-\frac{Li_3^2(-x)}{x}\Bigg |^1_0+\int^1_0\frac{2Li_2(-x)Li_3(-x)}{x^2}{\rm d}x=-\frac{9}{16}\zeta^2(3)-\frac{2Li_2(-x)Li_3(-x)}{x}\Bigg |^1_0+\int^1_0\frac{2Li_2^2(-x)}{x^2}{\rm d}x-\int^1_0\frac{2Li_3(-x)\ln(1+x)}{x^2}{\rm d}x

I=916ζ2(3)34ζ(2)ζ(3)2Li22(x)x01014Li2(x)ln(1+x)x2dx+2Li3(x)ln(1+x)x01012Li3(x)x(1+x)dx012Li2(x)ln(1+x)x2dx I=-\frac{9}{16}\zeta^2(3)-\frac{3}{4}\zeta(2)\zeta(3)-\frac{2Li_2^2(-x)}{x}\Bigg |^1_0-\int^1_0\frac{4Li_2(-x)\ln(1+x)}{x^2}{\rm d}x+\frac{2Li_3(-x)\ln(1+x)}{x}\Bigg |^1_0-\int^1_0\frac{2Li_3(-x)}{x(1+x)}{\rm d}x-\int^1_0\frac{2Li_2(-x)\ln(1+x)}{x^2}{\rm d}x

I=916ζ2(3)34ζ(2)ζ(3)12ζ2(2)+74ζ(4)32ζ(3)ln2+012Li3(x)1+xdx+6Li2(x) ln(1+x)x01016Li2(x)x(1+x)dx+016ln2(1+x)x2dx I=-\frac{9}{16}\zeta^2(3)-\frac{3}{4}\zeta(2)\zeta(3)-\frac{1}{2}\zeta^2(2)+\frac{7}{4}\zeta(4)-\frac{3}{2}\zeta(3)\ln{2}+\int^1_0\frac{2Li_3(-x)}{1+x}{\rm d}x+\frac{6Li_2(-x) \ \ln(1+x)}{x}\Bigg |^1_0-\int^1_0\frac{6Li_2(-x)}{x(1+x)}{\rm d}x+\int^1_0\frac{6\ln^2(1+x)}{x^2}{\rm d}x

I=916ζ2(3)34ζ(2)ζ(3)12ζ2(2)+74ζ(4)32ζ(3)ln2+9 2ζ(3)3ζ(2)ln2+016Li2(x)1+xdx+012Li3(x)1+xdx6ln2(1+x)x01+0112ln(1+x)x(1+x)dx I=-\frac{9}{16}\zeta^2(3)-\frac{3}{4}\zeta(2)\zeta(3)-\frac{1}{2}\zeta^2(2)+\frac{7}{4}\zeta(4)-\frac{3}{2}\zeta(3)\ln{2}+\frac{9} \ {2}\zeta(3)-3\zeta(2)\ln{2}+\int^1_0\frac{6Li_2(-x)}{1+x}{\rm d}x+\int^1_0\frac{2Li_3(-x)}{1+x}{\rm d}x-\frac{6\ln^2(1+x)}{x}\Bigg |^1_0+\int^1_0\frac{12\ln(1+x)}{x(1+x)}{\rm d}x

I=916ζ2(3)34ζ(2)ζ(3)12ζ2(2)+74ζ(4)32ζ(3)ln2+92ζ(3)3ζ(2)ln2+6ζ(2)12ln22+016Li2(x)1+xdx+012Li3(x)1+xdx I=-\frac{9}{16}\zeta^2(3)-\frac{3}{4}\zeta(2)\zeta(3)-\frac{1}{2}\zeta^2(2)+\frac{7}{4}\zeta(4)-\frac{3}{2}\zeta(3)\ln{2}+\frac{9}{2}\zeta(3)-3\zeta(2)\ln{2}+6\zeta(2)-12\ln^2{2}+\int^1_0\frac{6Li_2(-x)}{1+x}{\rm d}x+\int^1_0\frac{2Li_3(-x)}{1+x}{\rm d}x

Now,

I1=016Li2(x)1+xdx=6Li2(x)ln(1+x)01+016ln2(1+x)xdx I_1=\int^1_0\frac{6Li_2(-x)}{1+x}{\rm d}x=6Li_2(-x)\ln(1+x)\Bigg |^1_0+\int^1_0\frac{6\ln^2(1+x)}{x}{\rm d}x

I1=32ζ(3)3ζ(2)ln2I_1=\frac{3}{2}\zeta(3)-3\zeta(2)\ln{2}

I2=012Li3(x)1+xdx=2Li3(x)ln(1+x)01012Li2(x)ln(1+x)xdx2Li2(x)dLi2(x) I_2=\int^1_0\frac{2Li_3(-x)}{1+x}{\rm d}x=2Li_3(-x)\ln(1+x)\Bigg |^1_0-\int^1_0\underbrace{\frac{2Li_2(-x) \ln(1+x)}{x} {\rm d}x}_{\displaystyle\small{2Li_2(-x){\rm d}Li_2(-x)}}

I2=14ζ2(2)32ζ(3)ln2I_2=\frac{1}{4}\zeta^2(2)-\frac{3}{2}\zeta(3)\ln{2}

Therefore,

01Li32(x)x2dx=916ζ2(3)34ζ(2)ζ(3)14ζ2(2)+74ζ(4)3ζ(3)ln2+6ζ(3)6ζ(2)ln2+6ζ(2)12ln22\boxed{\displaystyle \int_0^1 \frac{{\rm Li}_3^2(-x)}{x^2}\, dx = -\frac{9}{16}\zeta^2(3)-\frac{3}{4}\zeta(2)\zeta(3)-\frac{1}{4}\zeta^2(2)+\frac{7}{4}\zeta(4)-3\zeta(3)\ln 2+6\zeta(3)-6\zeta(2)\ln 2+6\zeta(2)-12\ln^2 2}

Aditya Kumar - 4 years, 6 months ago

Problem 10:

Show that 01(x1)exlnxdx  =  e1e \int_0^1 (x-1) e^{-x} \ln x\,dx \; = \; \frac{e-1}{e}

This problem has been solved by Jasper Braun.

Mark Hennings - 4 years, 6 months ago

Log in to reply

Solution to Problem 10:

01xexlnxexlnxdx=01exlnxdx+01exdx01exlnxdx=11e (by IBP)\displaystyle\int_0^1xe^{-x}\ln{x}-e^{-x}\ln{x}dx = \int_0^1e^{-x}\ln{x}dx+\int_0^1e^{-x}dx-\int_0^1e^{-x}\ln{x}dx = \boxed{1-\frac{1}{e}}\text{ (by IBP)}

First Last - 4 years, 6 months ago

Log in to reply

You can also apply IBP this way (though almost same way),

01(1x)exlnx dx=01lnx d(xex) -\int_{0}^{1} (1-x)e^{-x} \ln x \ \mathrm{d}x = -\int_{0}^{1} \ln x \ \mathrm{d}(xe^{-x})

Ishan Singh - 4 years, 6 months ago

@Aditya Kumar @Aditya Sharma @Ishan Singh @Mark Hennings @Rohith M.Athreya and all other participants:

I have made this to make seeing old problems easier. I will complete it by tomorrow.

First Last - 4 years, 6 months ago

Log in to reply

That's very nice of you to compile the past problems! Thanks :)

Ishan Singh - 4 years, 6 months ago

Log in to reply

Sure! I hope it will make things easier!

First Last - 4 years, 6 months ago

really nice :)

thank you

Rohith M.Athreya - 4 years, 6 months ago

Problem 11:

Show that 01tα1tβ1(1+t)lntdt  =  ln(Γ(12+12α)Γ(12β)Γ(12+12β)Γ(12α)) \int_0^1 \frac{t^{\alpha-1} - t^{\beta-1}}{(1+t)\ln t}\,dt \; = \; \ln\left(\frac{\Gamma\big(\tfrac12+\tfrac12\alpha\big)\Gamma\big(\tfrac12\beta\big)}{\Gamma\big(\tfrac12+\tfrac12\beta\big)\Gamma\big(\tfrac12\alpha\big)}\right) for all α,β>0\alpha,\beta > 0.

This problem has been solved by Aditya Sharma.

Mark Hennings - 4 years, 6 months ago

Log in to reply

Results used : n01n+a=ψ(a)\displaystyle \sum_{n\ge 0}\frac{1}{n+a} =-\psi(a), ψ(ax)=1aln(Γ(ax))\displaystyle \int \psi(ax) = \frac{1}{a}\ln(\Gamma(ax))

Let F(a)=01xa1(1+x)lnxdx\displaystyle F(a)=\int_0^1 \frac{x^{a-1}}{(1+x)\ln x}dx then by differentiating w.r.t to a,

F(a)=01xa11+xdx=n0(1)n1n+a\displaystyle F'(a)=\int_0^1 \frac{x^{a-1}}{1+x}dx = \sum_{n\ge 0}(-1)^n \frac{1}{n+a}

Now , n0(1)n1n+a=12(n01n+a2n01n+a+12)=12(ψ(a+12)ψ(a2))\displaystyle \sum_{n\ge 0}(-1)^n \frac{1}{n+a} = \frac{1}{2}(\sum_{n\ge 0} \frac{1}{n+\frac{a}{2}}-\sum_{n\ge 0} \frac{1}{n+\frac{a+1}{2}}) = \frac{1}{2}(\psi(\frac{a+1}{2})-\psi(\frac{a}{2}))

So , F(α)F(β)=12(ψ(α+12)ψ(α2))12(ψ(β+12)ψ(β2))\displaystyle F'(\alpha)-F'(\beta) = \frac{1}{2}(\psi(\frac{\alpha+1}{2})-\psi(\frac{\alpha}{2}))-\frac{1}{2}(\psi(\frac{\beta+1}{2})-\psi(\frac{\beta}{2}))

Integrating we have, F(α)F(β)=(ln(Γ(1+α2)Γ(β2)Γ(1+β2)Γ(α2)))+C\displaystyle F(\alpha)-F(\beta) = \left(\ln(\frac{\Gamma(\frac{1+\alpha}{2})\Gamma(\frac{\beta}{2})}{\Gamma(\frac{1+\beta}{2})\Gamma(\frac{\alpha}{2})})\right)+C

Now since, F(1)=0    C=0F(1)=0\implies C=0

So F(α)F(β)=(lnΓ(1+α2)Γ(β2)Γ(1+β2)Γ(α2))\displaystyle F(\alpha)-F(\beta) = \left(\ln\frac{\Gamma(\frac{1+\alpha}{2})\Gamma(\frac{\beta}{2})}{\Gamma(\frac{1+\beta}{2})\Gamma(\frac{\alpha}{2})}\right)

Aditya Narayan Sharma - 4 years, 6 months ago

Problem 18 : Prove That

0log(a2+2acospx+1a2+2acosqx+1) dxx=2log(qp)log(1+a)  ; 1<a1 \int_{0}^{\infty} \log \left( \dfrac{a^2 + 2a \cos px + 1}{a^2 + 2a \cos qx + 1} \right) \ \dfrac{\mathrm{d}x}{x} = 2 \log \left( \dfrac{q}{p} \right) \log(1+a) \ \ ; \ -1 < a \leq 1

This problem has been solved by Mark Hennings.

Ishan Singh - 4 years, 5 months ago

Log in to reply

There is some delicate interchanging of integrals and infinite sums needed here, so I will set this out in full. Fix p,q>0p,q > 0. For 0<ε<X0 < \varepsilon < X we have εXcoskpxcoskqxxdx  =  (pεpXqεqX)coskxxdx  =  qXpXcoskxxdxqεpεcoskxxdx \int_{\varepsilon}^X \frac{\cos^k px - \cos^k qx}{x}\,dx \; =\; \left(\int_{p\varepsilon}^{pX} - \int_{q\varepsilon}^{qX}\right) \frac{\cos^kx}{x}\,dx \; =\; \int_{qX}^{pX} \frac{\cos^kx}{x}\,dx - \int_{q\varepsilon}^{p\varepsilon}\frac{\cos^kx}{x}\,dx which implies that εXcoskpqcoskqxxdx    2pqmin(p,q)0<ε<X \left|\int_{\varepsilon}^X \frac{\cos^k pq - \cos^k qx}{x}\,dx \right| \; \le \; \frac{2|p-q|}{\mathrm{min}(p,q)} \hspace{1cm} 0 < \varepsilon < X Thus, if we define the functions fk(X)  =  0XcoskpxcoskqxxdxkN,X>0 f_k(X) \; = \; \int_0^X \frac{\cos^k px - \cos^k qx}{x}\,dx \hspace{2cm} k \in \mathbb{N}\,,\, X > 0 then we have shown that fk(X)    2pqmin(p,q)kN,X>0 \big|f_k(X)\big| \; \le \; \frac{2|p-q|}{\mathrm{min}(p,q)} \hspace{2cm} k \in \mathbb{N}\,,\, X > 0 and hence the functions fk(X)f_k(X) are uniformly bounded in both kk and XX. Moreover, the well-known Frullani integrals 0cosaxcosbxxdx  =  ln(ba)0sinaxsinbxxdx  =  0 \int_0^\infty \frac{\cos ax - \cos bx}{x}\,dx \; = \; \ln\big(\tfrac{b}{a}\big) \hspace{2cm} \int_0^\infty \frac{\sin ax - \sin bx}{x}\,dx \; = \; 0 which hold whenever aa and bb are nonzero with the same sign, imply that 0eirpxeirqxxdx  =  ln(qp)r0 \int_0^\infty \frac{e^{irpx} - e^{irqx}}{x}\,dx \; =\; \ln\big(\tfrac{q}{p}\big) \hspace{2cm} r \neq 0 and hence, since fk(X)  =  2k0X(eipx+eipx)k(eiqx+eiqx)kxdx  =  2kr=0k(kr)0Xei(2rk)pxei(2rk)qxxdx f_k(X) \; = \; 2^{-k} \int_0^X \frac{(e^{ipx} + e^{ipx})^k - (e^{iqx} + e^{-iqx})^k}{x}\,dx \; = \; 2^{-k} \sum_{r=0}^k {k \choose r}\int_0^X \frac{e^{i(2r-k)px} - e^{i(2r-k)qx}}{x}\,dx we deduce that φk  =  limXfk(X)  =  {2kr=0k(kr)ln(qp)k odd2k0rkr12k(kr)ln(qp)k even \varphi_k \; = \; \lim_{X \to \infty} f_k(X) \; = \; \left\{ \begin{array}{ll} \displaystyle 2^{-k} \sum_{r=0}^k {k \choose r} \ln\big(\tfrac{q}{p}\big) & k \mbox{ odd} \\ \displaystyle 2^{-k} \sum_{0 \le r \le k \atop r \neq \frac12k} {k \choose r}\ln\big(\tfrac{q}{p}\big) & k \mbox{ even} \end{array} \right. and hence that φk  =  {ln(qp)k oddln(qp)(12k(k12k))k even \varphi_k \; = \; \left\{ \begin{array}{ll} \ln\big(\tfrac{q}{p}\big) & k \mbox{ odd} \\ \ln\big(\tfrac{q}{p}\big)\left(1 - 2^{-k}{k \choose \frac12k}\right) & k \mbox{ even} \end{array} \right. Now, for 1<a<1-1 < a < 1, 0Xln(1+2acospx+a21+2acosqx+a2)dxx=0X{ln(1+2a1+a2cospx)ln(1+2a1+a2cosqx)}dxx=0Xk=1(1)k1k(2aa2+1)k[coskpxcoskqx]dxx=k=1(1)k1k(2aa2+1)kfk(X)\begin{aligned} \int_0^X \ln\left(\frac{1 + 2a \cos px + a^2}{1 + 2a \cos qx + a^2}\right)\,\frac{dx}{x} & = \int_0^X \left\{ \ln\big(1 + \tfrac{2a}{1+a^2}\cos px\big) -\ln\big(1 + \tfrac{2a}{1+a^2}\cos qx\big)\right\}\,\frac{dx}{x} \\ & = \int_0^X \sum_{k=1}^\infty \frac{(-1)^{k-1}}{k}\big(\tfrac{2a}{a^2+1}\big)^k\big[\cos^k px - \cos^k qx\big]\,\frac{dx}{x} \\ & = \sum_{k=1}^\infty \frac{(-1)^{k-1}}{k}\big(\tfrac{2a}{a^2+1}\big)^k f_k(X) \end{aligned} where the interchange of the sum and integral in the last line is valid because 2aa2+1<1\big|\tfrac{2a}{a^2+1}\big| < 1, and hence the series is uniformly convergent in xx. We can now use the uniform boundedness of the functions fk(X)f_k(X) to deduce that 0ln(1+2acospx+a21+2acosqx+a2)dxx=limX0Xln(1+2acospx+a21+2acosqx+a2)dxx=k=1(1)k1k(2aa2+1)kφk \begin{aligned} \int_0^\infty \ln\left(\frac{1 + 2a \cos px + a^2}{1 + 2a \cos qx + a^2}\right)\,\frac{dx}{x} & = \lim_{X \to \infty}\int_0^X \ln\left(\frac{1 + 2a \cos px + a^2}{1 + 2a \cos qx + a^2}\right)\,\frac{dx}{x} \\ & = \sum_{k=1}^\infty \frac{(-1)^{k-1}}{k}\big(\tfrac{2a}{a^2+1}\big)^k \varphi_k \end{aligned} Thus the integral is equal to ln(qp)[k=1(1)k1k(2aa2+1)k+k=112k(2aa2+1)2k22k(2kk)]=ln(qp)[ln(1+2aa2+1)+12k=11k(2kk)(a2(a2+1)2)k]=ln(qp)[ln((1+a)2a2+1)ln(12[1+14a2(a2+1)2])]=2ln(qp)ln(1+a)\begin{aligned} \ln\big(\tfrac{q}{p}\big)&\left[ \sum_{k=1}^\infty \frac{(-1)^{k-1}}{k}\big(\tfrac{2a}{a^2+1}\big)^k + \sum_{k=1}^\infty \frac{1}{2k}\big(\tfrac{2a}{a^2+1}\big)^{2k} 2^{-2k}{2k \choose k}\right] \\ & = \ln\big(\tfrac{q}{p}\big)\left[\ln\big(1 + \tfrac{2a}{a^2+1}\big) + \tfrac12\sum_{k=1}^\infty \frac{1}{k}{2k \choose k} \big(\tfrac{a^2}{(a^2+1)^2}\big)^{k}\right] \\ & = \ln\big(\tfrac{q}{p}\big)\left[\ln\big(\tfrac{(1+a)^2}{a^2+1}\big) - \ln\left(\tfrac12\left[1 + \sqrt{1 -4\tfrac{a^2}{(a^2+1)^2}}\right]\right)\right] \\ & = 2\ln\big(\tfrac{q}{p}\big)\ln(1+a) \end{aligned} as required.

Mark Hennings - 4 years, 5 months ago

Log in to reply

(+1) Nice solution! There is a slight typo in the Frullani Integral for cosine, the integral is for aa and bb whereas the logarithm is for pp and qq.

Ishan Singh - 4 years, 5 months ago

Problem 19:

Show that 0expexqxdx  =  pqpqγ \int_0^\infty \frac{e^{-x^p} - e^{-x^q}}{x}\,dx \; = \; \frac{p-q}{pq}\gamma for p,q>0p,q > 0, where γ\gamma is the Euler-Mascheroni constant.

The problem has been solved first by Aditya Sharma and then by Ishan Singh.

Mark Hennings - 4 years, 5 months ago

Log in to reply

Solution to Problem 19

I=0(expexq)d(lnx)\displaystyle I=\int_0^\infty (e^{-x^p}-e^{-x^q})d(\ln x)

I=[(expexq)lnx]0+p0expxp1lnxq0exqxq1lnxdx\displaystyle I= [(e^{-x^p}-e^{-x^q})\ln x]_0^\infty + p\int_0^\infty e^{-x^p}x^{p-1}\ln x-q\int_0^\infty e^{-x^q}x^{q-1}\ln x\,dx

The substitutions xp=u  ,xq=vx^p=u\;,x^q=v makes the last two integrals as,

p0expxp1lnxq0exqxq1lnxdx=0exxlnxdx(qpqp)\displaystyle p\int_0^\infty e^{-x^p}x^{p-1}\ln x-q\int_0^\infty e^{-x^q}x^{q-1}\ln x\,dx = \int_0^\infty e^{-x}x\ln x\, dx \left(\frac{q-p}{qp}\right)

=0exlnxdx(qpqp)=γ(pqqp)\displaystyle = \int_0^\infty e^{-x}\ln x\, dx \left(\frac{q-p}{qp}\right) = \gamma\left(\frac{p-q}{qp}\right)

Now [(expexq)lnx]0=limxlnx1expexqlimx0lnx1expexq\displaystyle [(e^{-x^p}-e^{-x^q})\ln x]_0^\infty = \lim_{x\to\infty} \frac{\ln x}{\frac{1}{e^{-x^p}-e^{-x^q}}}- \lim_{x\to 0} \frac{\ln x}{\frac{1}{e^{-x^p}-e^{-x^q}}}

Both of these can be evaluated by applying L-Hospital's rule succesively and both of them are 00.

Hence 0(expexq)xdx=γ(pqpq)\displaystyle \int_0^\infty \frac{(e^{-x^p}-e^{-x^q})}{x}\, dx = \gamma\left(\frac{p-q}{pq}\right)

Aditya Narayan Sharma - 4 years, 5 months ago

Solution to Problem 19 :

Proposition 1 : 0(ex1x+1)dxx=γ \int_{0}^{\infty} \left(e^{-x} - \dfrac{1}{x+1}\right)\dfrac{\mathrm{d}x}{x} = -\gamma

Proof : Let,

J=0(ex1x+1)dxx \text{J} = \int_{0}^{\infty} \left(e^{-x} - \dfrac{1}{x+1}\right)\dfrac{\mathrm{d}x}{x}

=00ex(a+1)eaxx+1dx da = \int_{0}^{\infty} \int_{0}^{\infty} e^{-x(a+1)} - \dfrac{e^{-ax}}{x+1} \mathrm{d}x \ \mathrm{d}a

=01a+1da00eaxx+1dx da\displaystyle = \int_{0}^{\infty} \dfrac{1}{a+1} \mathrm{d}a -\int_{0}^{\infty}\int_{0}^{\infty} \dfrac{e^{-ax}}{x+1}\mathrm{d}x \ \mathrm{d}a

Substitute axxax \mapsto x in the second integral,

    J=01a+1da00exx+adx da\displaystyle \implies \text{J} = \int_{0}^{\infty} \dfrac{1}{a+1} \mathrm{d}a -\int_{0}^{\infty}\int_{0}^{\infty} \dfrac{e^{-x}}{x+a}\mathrm{d}x \ \mathrm{d}a

Since 0exdx=1\displaystyle \int_{0}^{\infty} e^{-x}\mathrm{d}x = 1

    J=00exa+1exx+adx da\displaystyle \implies \text{J} = \int_{0}^{\infty} \int_{0}^{\infty} \dfrac{e^{-x}}{a+1} - \dfrac{e^{-x}}{x+a}\mathrm{d}x \ \mathrm{d}a

=0ex01a+11x+ada dx\displaystyle = \int_{0}^{\infty} e^{-x} \int_{0}^{\infty} \dfrac{1}{a+1} - \dfrac{1}{x+a}\mathrm{d}a \ \mathrm{d}x

=0ex[ln(a+1a+x)]a=0adx\displaystyle = \int_{0}^{\infty} e^{-x} \left[\ln\left(\dfrac{a+1}{a+x}\right)\right]_{a=0}^{a \to \infty} \mathrm{d}x

=0exlnx dx\displaystyle =\int_{0}^{\infty} e^{-x} \ln x \ \mathrm{d}x

=γ\displaystyle = -\gamma \quad \square

Proposition 2 : 011exxdx1exxdx=γ\displaystyle \int_{0}^{1} \dfrac{1-e^{-x}}{x} \mathrm{d}x - \int_{1}^{\infty}\dfrac{e^{-x}}{x} \mathrm{d}x = \gamma

Proof : 011exxdx1exxdx=001(1ex)eyxdx dy01ex(y+1)dx dy\displaystyle \int_{0}^{1} \dfrac{1-e^{-x}}{x} \mathrm{d}x - \int_{1}^{\infty}\dfrac{e^{-x}}{x} \mathrm{d}x = \int_{0}^{\infty} \int_{0}^{1} (1-e^{-x})e^{-yx} \mathrm{d}x \ \mathrm{d}y - \int_{0}^{\infty} \int_{1}^{\infty} e^{-x(y+1)} \mathrm{d}x \ \mathrm{d}y

=0(1eyy+e(y+1)1y+1)dy0e(y+1)y+1dy\displaystyle = \int_{0}^{\infty} \left(\dfrac{1-e^{-y}}{y} + \dfrac{e^{-(y+1)} - 1}{y+1} \right) \mathrm{d}y - \int_{0}^{\infty} \dfrac{e^{(y+1)}}{y+1} \mathrm{d}y

=0(ey1y+1)dyy\displaystyle = -\int_{0}^{\infty} \left(e^{-y} - \dfrac{1}{y+1} \right)\dfrac{\mathrm{d}y}{y}

=γ (Using Proposition 1)  = \gamma \ \left( \text{Using Proposition 1} \right) \ \square

Now,

I=0expexqx dx\text{I} = \int_{0}^{\infty} \dfrac{e^{-x^p} - e^{-x^q}}{x} \ \mathrm{d}x

=(011exqx dx1exqx dx)(011expx dx1expx dx) = \left( \int_{0}^{1} \dfrac{1-e^{-x^q}}{x} \ \mathrm{d}x - \int_{1}^{\infty} \dfrac{e^{-x^q}}{x} \ \mathrm{d}x \right) - \left( \int_{0}^{1} \dfrac{1-e^{-x^p}}{x} \ \mathrm{d}x - \int_{1}^{\infty} \dfrac{e^{-x^p}}{x} \ \mathrm{d}x \right)

Substitute xqx x^q \mapsto x in the first integral and xpxx^p \mapsto x in the second integral to get,

I=1q(011exx dx1exx dx)1p(011exx dx1exx dx) \text{I} = \dfrac{1}{q} \left( \int_{0}^{1} \dfrac{1-e^{-x}}{x} \ \mathrm{d}x - \int_{1}^{\infty} \dfrac{e^{-x}}{x} \ \mathrm{d}x \right) - \dfrac{1}{p} \left( \int_{0}^{1} \dfrac{1-e^{-x}}{x} \ \mathrm{d}x - \int_{1}^{\infty} \dfrac{e^{-x}}{x} \ \mathrm{d}x \right)

Using Proposition 2, we get,

I=(1q1p)γ \text{I} = \left(\dfrac{1}{q} - \dfrac{1}{p}\right) \gamma

I=pqpqγ \therefore \text{I} = \dfrac{p-q}{pq} \gamma \quad \square

Ishan Singh - 4 years, 5 months ago

Problem 20 :

Show that 01xlnxarctanx1+xdx=12ln212Gln2+π364+π248π4\displaystyle \int_0^1 \dfrac{x\ln x\arctan x}{1+x}\, dx=\dfrac{1}{2}\ln 2-\dfrac{1}{2}G\ln 2+\dfrac{\pi^3}{64}+\dfrac{\pi^2}{48}-\dfrac{\pi}{4}.

This problem has been solved by Mark Hennings and then by Fdp Dpf.

Aditya Narayan Sharma - 4 years, 5 months ago

Log in to reply

Note that I  =  01xlnxtan1x1+xdx=01xlnx1+x(0xdu1+u2)dx=01(u1xlnx1+xdx)du1+u2=01[xlnxxlnxln(1+x)Li2(x)]x=u1du1+u2=01(1+112π2ulnu+u+lnuln(1+u)+Li2(u)]du1+u2=14π+148π2+148π3+12ln2+01lnuln(1+u)+Li2(u)1+u2du\begin{aligned} I \; = \; \int_0^1 \frac{x \,\ln x\, \tan^{-1}x}{1+x}\,dx & = \int_0^1 \frac{x \ln x}{1+x}\left(\int_0^x \frac{du}{1+u^2}\right)\,dx \\ & = \int_0^1 \left(\int_u^1 \frac{x \ln x}{1+x}\,dx \right)\,\frac{du}{1+u^2} \\ & = \int_0^1 \Big[x\ln x - x - \ln x \ln(1+x) - \mathrm{Li}_2(-x)\Big]_{x=u}^1\,\frac{du}{1+u^2} \\ & = \int_0^1 \Big(-1 + \tfrac{1}{12}\pi^2 - u\ln u + u + \ln u \ln(1+u) +\mathrm{Li}_2(-u)\Big]\,\frac{du}{1+u^2} \\ & = -\tfrac14\pi + \tfrac{1}{48}\pi^2 + \tfrac{1}{48}\pi^3 + \tfrac12\ln2 + \int_0^1 \frac{\ln u \ln(1+u) + \mathrm{Li}_2(-u)}{1 + u^2}\,du \end{aligned} It is possible (integrating by parts and so forth) to calculate the indefinite integrals of both parts of the remaining integral in terms of polylogarithms, but the end result is extremely complicated! My expression for the indefinite integral lnuln(1+u)1+u2du \int \frac{\ln u \ln(1+u)}{1 + u^2}\,du has over 3030 terms, and the indefinite integral for Li2(u)1+u2du \int \frac{\mathrm{Li}_2(-u)}{1+u^2}\,du is even more complicated. The definite integrals are 01lnuln(1+u)1+u2du=1128(11π3256Gln2+10iπ2ln2+12π(ln2)28i(ln2)3394iLi3(12+12i)+210ζ(3))01Li2(u)1+u2du=1384(35π330iπ2ln236π(ln2)2+576Gln2+24i(ln2)3768iLi3(12+12i)+630iζ(3))\begin{aligned} \int_0^1 \frac{\ln u \ln(1+u)}{1 + u^2}\,du & = \tfrac{1}{128} \left(\begin{array}{l}11 \pi^3 - 256 G \ln2 + 10 i \pi^2 \ln2 + 12\pi (\ln2)^2 \\ - 8i(\ln2)^3 - 394i\mathrm{Li}_3(\tfrac12+\tfrac12i) + 210\zeta(3)\end{array}\right) \\ \int_0^1 \frac{\mathrm{Li}_2(-u)}{1+u^2}\,du & = \tfrac{1}{384}\left(\begin{array}{l}-35\pi^3 - 30 i \pi^2 \ln2 - 36 \pi (\ln2)^2 + 576G \ln2\\ + 24i(\ln2)^3 - 768i\mathrm{Li}_3(\tfrac12+\tfrac12i) + 630i\zeta(3)\end{array}\right) \end{aligned} so that 01lnxln(1+x)+Li2(x)1+x2dx  =  1192π312Gln2 \int_0^1 \frac{\ln x \ln(1+x) + \mathrm{Li}_2(-x)}{1 + x^2}\,dx \; = \; -\tfrac{1}{192}\pi^3 - \tfrac12 G \ln2 and hence I  =  14π+148π2+164π3+12ln212Gln2 I \; = \; -\tfrac14\pi + \tfrac{1}{48}\pi^2 + \tfrac{1}{64}\pi^3 + \tfrac12\ln2 - \tfrac12G \ln2 as required.

That was exhausting! Someone else can set the next one...

Mark Hennings - 4 years, 5 months ago

SOLUTION OF PROBLEM 20:

Alternative solution.

Let R(x)=0xtlnt1+tdt=01tx2ln(tx)1+xtdt\displaystyle R(x)=\int_0^x \dfrac{t\ln t}{1+t}dt=\int_0^1 \dfrac{tx^2\ln(tx)}{1+xt}dt

Note that,

R(1)=π2121R(1)=\dfrac{\pi^2}{12}-1

(Taylor expansion of integrand)

I=01xlnxarctanx1+xdx=[R(x)arctanx]0101R(x)1+x2dx=π348π40101tx2ln(tx)(1+tx)(1+x2)dtdx=π348π40101tx2lnt(1+tx)(1+x2)dtdx0101tx2lnx(1+tx)(1+x2)dtdx=π348π401[tlnt(tln(1+x2)2(1+t2)+ln(1+tx)t(t2+1)arctanx1+t2)]x=0x=1dt01[lnx(txln(1+tx))1+x2]t=0t=1dx=π348π4ln2201t2lnt1+t2dt01lntln(1+t)1+t2dt+π401tlnt1+t2dt01xlnx1+x2dx+01lnxln(1+x)1+x2dx=π348π4ln2201t2lnt1+t2dt+(π41)01tlnt1+t2dt\begin{aligned} \displaystyle I=\int_0^1 \dfrac{x\ln x \arctan x}{1+x}dx \displaystyle &=\Big[R(x)\arctan x\Big]_0^1 -\int_0^1 \dfrac{R(x)}{1+x^2}dx\\ \displaystyle &= \dfrac{\pi^3}{48}-\dfrac{\pi}{4}-\int_0^1\int_0^1 \dfrac{tx^2\ln(tx)}{(1+tx)(1+x^2)}dtdx\\ \displaystyle &= \dfrac{\pi^3}{48}-\dfrac{\pi}{4}-\int_0^1\int_0^1 \dfrac{tx^2\ln t}{(1+tx)(1+x^2)}dtdx-\int_0^1\int_0^1 \dfrac{tx^2\ln x}{(1+tx)(1+x^2)}dtdx\\ &\displaystyle =\dfrac{\pi^3}{48}-\dfrac{\pi}{4}-\int_0^1 \left[t\ln t\left(\dfrac{t\ln(1+x^2)}{2(1+t^2)}+\dfrac{\ln(1+tx)}{t(t^2+1)}-\dfrac{\arctan x}{1+t^2}\right)\right]_{x=0}^{x=1} dt-\\ &\displaystyle \int_0^1 \left[\dfrac{\ln x\Big(tx-\ln(1+tx)\Big)}{1+x^2}\right]_{t=0}^{t=1}dx\\ &=\displaystyle\dfrac{\pi^3}{48}-\dfrac{\pi}{4}-\dfrac{\ln 2}{2}\int_0^1 \dfrac{t^2\ln t}{1+t^2}dt-\int_0^1 \dfrac{\ln t\ln(1+t)}{1+t^2}dt+\dfrac{\pi}{4}\int_0^1 \dfrac{t\ln t}{1+t^2}dt-\int_0^1 \dfrac{x\ln x}{1+x^2}dx+\\ &\int_0^1 \dfrac{\ln x\ln(1+x)}{1+x^2}dx\\ &=\displaystyle \dfrac{\pi^3}{48}-\dfrac{\pi}{4}-\dfrac{\ln 2}{2}\int_0^1 \dfrac{t^2\ln t}{1+t^2}dt+\left(\dfrac{\pi}{4}-1\right)\int_0^1 \dfrac{t\ln t}{1+t^2}dt\\ \end{aligned}

Using Taylor expansion ofx2lnx1+x2\dfrac{x^2\ln x}{1+x^2} and xlnx1+x2\dfrac{x\ln x}{1+x^2} one obtains,

01x2lnx1+x2dx=G1\displaystyle \int_0^1 \dfrac{x^2\ln x}{1+x^2}dx=G-1

G being the Catalan constant.

01xlnx1+x2dx=π248\displaystyle \int_0^1 \dfrac{x\ln x}{1+x^2}dx=-\dfrac{\pi^2}{48}

Therefore,

I=π364+π248π412Gln2+12ln2\boxed{\displaystyle I=\dfrac{\pi^3}{64}+\dfrac{\pi^2}{48}-\dfrac{\pi}{4}-\dfrac{1}{2}G\ln 2+\dfrac{1}{2}\ln 2}

FDP DPF - 4 years, 5 months ago

Log in to reply

(+1) Nice solution! You might be interested in Part 2 of the current season.

Ishan Singh - 4 years, 5 months ago

Problem 7:

Show that for n2n\ge2, 0lnxxn+1dx=π2n2cot(πn)csc(πn)\displaystyle \int_0^\infty \frac{\ln x}{x^n+1}dx = -\frac{\pi^2}{n^2}\cot\left(\frac{\pi}{n}\right)\csc\left(\frac{\pi}{n}\right)

This problem has been solved by Mark Hennings.

Aditya Narayan Sharma - 4 years, 6 months ago

Log in to reply

Solution to Problem 7:

Consider the integrals of f(z)  =  1zn+1g(z)  =  logzzn+1 f(z) \; = \; \frac{1}{z^n + 1} \hspace{1cm} g(z) \; = \; \frac{\log z}{z^n + 1} around the contour γ1+γ2γ3\gamma_1 + \gamma_2 - \gamma_3, where

  • γ1\gamma_1 is the line segment z=xz \,=\, x, for 0<x<R0 < x < R,
  • γ2\gamma_2 is the circular contour z=Reiθz \,=\, Re^{i\theta} for 0<θ<2πn0 < \theta < \frac{2\pi}{n},
  • γ3\gamma_3 the the line segment z=xe2πinz \,=\, x e^{\frac{2\pi i}{n}}, for 0<x<R0 < x < R.

where R>1R > 1. Now γ1f(z)dz=0R1xn+1dxγ3f(z)dz=e2πin0R1xn+1dx(γ1γ3)f(z)dz=2ieπinsinπn0R1xn+1dxγ1g(z)dz=0Rlnxxn+1dxγ3g(z)dz=e2πin0Rlnx+iπnxn+1dx(γ1γ3)g(z)dz=2ieπinsinπn0Rlnxxn+1dxiπne2πin0R1xn+1dx\begin{aligned} \int_{\gamma_1} f(z)\,dz & = \int_0^R \frac{1}{x^n+1}\,dx \\ \int_{\gamma_3} f(z)\,dz & = e^{\frac{2 \pi i}{n}}\int_0^R \frac{1}{x^n + 1}\,dx \\ \left(\int_{\gamma_1} - \int_{\gamma_3}\right)\,f(z)\,dz & = -2ie^{\frac{\pi i}{n}}\sin\frac{\pi}{n}\int_0^R \frac{1}{x^n + 1}\,dx \\ \int_{\gamma_1} g(z)\,dz & = \int_0^R \frac{\ln x}{x^n+1}\,dx \\ \int_{\gamma_3} g(z)\,dz & = e^{\frac{2 \pi i}{n}}\int_0^R \frac{\ln x + i\frac{\pi}{n}}{x^n + 1}\,dx \\ \left(\int_{\gamma_1} - \int_{\gamma_3}\right)\,g(z)\,dz & = -2ie^{\frac{\pi i}{n}}\sin\frac{\pi}{n}\int_0^R \frac{\ln x}{x^n + 1}\,dx - i\frac{\pi}{n}e^{\frac{2\pi i}{n}}\int_0^R \frac{1}{x^n + 1}\,dx \end{aligned} Since γ2f(z)dz  =  O(R1n)γ2g(z)dz  =  O(lnRR1n) \int_{\gamma_2} f(z)\,dz \; = \; O\big(R^{1-n}\big) \hspace{2cm} \int_{\gamma_2} g(z)\,dz \; = \; O\big(\ln R \,R^{1-n}\big) as RR \to \infty, we deduce that (since both f(z)f(z) and g(z)g(z) just have simple poles at eπine^{\frac{\pi i}{n}} inside this contour) 2ieπinsinπn01xn+1dx=2πiResz=eπinf(z)  =  2πineπin2ieπinsinπn0Rlnxxn+1dxiπne2πin0R1xn+1dx=2πiResz=eπing(z)  =  2π2n2eπin \begin{aligned} \displaystyle -2ie^{\frac{\pi i}{n}}\sin\frac{\pi}{n}\int_0^\infty \frac{1}{x^n + 1}\,dx & = \displaystyle 2\pi i \mathrm{Res}_{z = e^{\frac{\pi i}{n}}} f(z) \; = \; -\frac{2\pi i}{n}e^{\frac{\pi i}{n}} \\ \displaystyle -2ie^{\frac{\pi i}{n}}\sin\frac{\pi}{n}\int_0^R \frac{\ln x}{x^n + 1}\,dx - i\frac{\pi}{n}e^{\frac{2\pi i}{n}}\int_0^R \frac{1}{x^n + 1}\,dx & =\displaystyle 2\pi i \mathrm{Res}_{z = e^{\frac{\pi i}{n}}} g(z) \; = \; \frac{2\pi ^2}{n^2}e^{\frac{\pi i}{n}} \end{aligned} and so 01xn+1dx  =  πncosecπn0lnxxn+1dx  =  π2n2cotπncosecπn \int_0^\infty \frac{1}{x^n + 1}\,dx \; = \; \frac{\pi}{n}\mathrm{cosec}\,\frac{\pi}{n} \hspace{2cm} \int_0^\infty \frac{\ln x}{x^n + 1}\,dx \; = \; -\frac{\pi^2}{n^2}\, \cot\frac{\pi}{n}\,\mathrm{cosec}\frac{\pi}{n}

Mark Hennings - 4 years, 6 months ago

Problem 9:

Prove 01Li22(x)dx=π4364ζ(3)2ζ(2)+6\displaystyle\int_0^1Li_2^2(x)dx = \frac{\pi^4}{36}-4\zeta(3)-2\zeta(2)+6

This problem has been solved by Mark Hennings.

First Last - 4 years, 6 months ago

Log in to reply

I will be using the Euler result that u=1Huu2  =  2ζ(3) \sum_{u=1}^\infty\frac{H_u}{u^2} \; = \; 2\zeta(3) Integrating by parts, 01Li22(x)dx=ζ(2)2201Li1(x)Li2(x)dx  =  ζ(2)22r,s=11rs201xr+sdx=ζ(2)22r,s=11rs2(r+s+1)  =  ζ(2)22r,s=11s2(s+1)(1r1r+s+1)=ζ(2)22s=1Hs+1s2(s+1)  =  ζ(2)22s=1Hs+1(1s21s(s+1))=ζ(2)22s=1Hs+1s2+2s=1Hs+1s(s+1)=ζ(2)22s=1Hss22s=11s2(s+1)+2s=1Hs+1(1s1s+1)=ζ(2)24ζ(3)2s=1(1s21s(s+1))+2limS(s=1SHs+1ss=2S+1Hss)=ζ(2)24ζ(3)2ζ(2)+2+2limS(32+s=2S1s(s+1)HS+1S)=ζ(2)24ζ(3)2ζ(2)+6 \begin{aligned} \int_0^1 \mathrm{Li}_2^2(x)\,dx & = \zeta(2)^2 - 2\int_0^1 \mathrm{Li}_1(x)\mathrm{Li}_2(x)\,dx \; = \; \zeta(2)^2 - 2\sum_{r,s=1}^\infty \frac{1}{rs^2}\int_0^1 x^{r+s}\,dx \\ & = \zeta(2)^2 - 2\sum_{r,s=1}^\infty \frac{1}{rs^2(r+s+1)} \; = \; \zeta(2)^2 - 2\sum_{r,s=1}^\infty\frac{1}{s^2(s+1)}\left(\frac{1}{r} - \frac{1}{r+s+1}\right) \\ &= \zeta(2)^2 - 2\sum_{s=1}^\infty \frac{H_{s+1}}{s^2(s+1)} \; = \; \zeta(2)^2 - 2\sum_{s=1}^\infty H_{s+1}\left(\frac{1}{s^2} - \frac{1}{s(s+1)}\right) \\ &= \zeta(2)^2 - 2\sum_{s=1}^\infty \frac{H_{s+1}}{s^2} + 2\sum_{s=1}^\infty \frac{H_{s+1}}{s(s+1)} \\ & = \zeta(2)^2 - 2\sum_{s=1}^\infty \frac{H_s}{s^2} - 2\sum_{s=1}^\infty \frac{1}{s^2(s+1)} + 2\sum_{s=1}^\infty H_{s+1}\left(\frac{1}{s} - \frac{1}{s+1}\right) \\ & = \zeta(2)^2 - 4\zeta(3) - 2\sum_{s=1}^\infty \left(\frac{1}{s^2} - \frac{1}{s(s+1)}\right) + 2\lim_{S \to \infty}\left(\sum_{s=1}^S \frac{H_{s+1}}{s} - \sum_{s=2}^{S+1} \frac{H_s}{s}\right) \\ &= \zeta(2)^2 - 4\zeta(3) - 2\zeta(2) + 2 + 2\lim_{S \to \infty}\left(\tfrac32 + \sum_{s=2}^S \frac{1}{s(s+1)} - \frac{H_{S+1}}{S}\right) \\ &= \zeta(2)^2 - 4\zeta(3) - 2\zeta(2) + 6 \end{aligned} as required.

Mark Hennings - 4 years, 6 months ago

Problem 12

Show that 0(lnxarctanxx)2dx=π3ln24π312+2πln2π4ζ(3)\displaystyle \int_0^\infty \left(\frac{\ln x\arctan x}{x}\right)^2 dx = \pi^3\frac{\ln 2}{4}-\frac{\pi^3}{12}+2\pi\ln 2-\frac{\pi}{4}\zeta(3)

This problem has been solved by Mark Hennings

Aditya Narayan Sharma - 4 years, 6 months ago

Log in to reply

Consider the principal branch of the logarithm, so that the plane is cut along the negative real axis. Suppose that a>0a > 0. If we integrate f(z)  =  (logz)2z2+a2 f(z) \; = \; \frac{(\log z)^2}{z^2 + a^2} around the contour γ1+γ2γ3γ4\gamma_1 + \gamma_2 - \gamma_3 - \gamma_4, where

  • γ1\gamma_1 is the straight line z=xz = x for ε<x<R\varepsilon < x < R,
  • γ2\gamma_2 is the semicircular arc z=Reiθz = Re^{i\theta} for 0<θ<π0 < \theta < \pi,
  • γ3\gamma_3 is the straight line z=xeiπz = xe^{i\pi} for ε<x<R\varepsilon < x < R, just above the cut,
  • γ4\gamma_4 is the semicircular arc z=εeiθz = \varepsilon e^{i\theta} for 0<θ<π0 < \theta < \pi,

where 0<ε<a<R0 < \varepsilon < a < R. Then γ1f(z)dz  =  εRln2xx2+a2dxγ3f(z)dz  =  εR(lnx+iπ)2x2+a2dx \int_{\gamma_1} f(z)\,dz \; = \; \int_\varepsilon^R \frac{\ln^2x}{x^2 + a^2}\,dx \hspace{2cm} \int_{\gamma_3} f(z)\,dz \; = \; -\int_\varepsilon^R \frac{(\ln x + i\pi)^2}{x^2 + a^2}\,dx while limε0γ4f(z)dz  =  limRγ2f(z)dz  =  0 \lim_{\varepsilon \to 0}\int_{\gamma_4} f(z)\,dz \; = \; \lim_{R \to \infty}\int_{\gamma_2} f(z)\,dz \; = \; 0 and hence we deduce that 0ln2x+(lnx+iπ)2x2+a2dx  =  2πiResz=iaf(z)  =  πa(lna+12πi)2 \int_0^\infty \frac{\ln^2x + (\ln x + i\pi)^2}{x^2+a^2}\,dx \; = \; 2\pi i \mathrm{Res}_{z=ia} f(z) \; = \; \tfrac{\pi}{a}(\ln a + \tfrac12\pi i)^2 so that 0ln2xx2+a2dx  =  π2aln2a+π38a0lnxx2+a2dx  =  π2alna \int_0^\infty \frac{\ln^2x}{x^2 + a^2}\,dx \; = \; \tfrac{\pi}{2a}\,\ln^2a + \tfrac{\pi^3}{8a} \hspace{2cm} \int_0^\infty \frac{\ln x}{x^2 + a^2}\,dx \; = \; \tfrac{\pi}{2a}\ln a

Now tan1xx  =  01du1+x2u2 \frac{\tan^{-1}x}{x} \; =\; \int_0^1 \frac{du}{1 + x^2u^2} and hence 0(tan1xlnxx)2dx=0101(0ln2x(1+x2u2)(1+x2v2)dx)dudv=0101(0ln2x{1x2+u21x2+v2}dx)dudvu2v2=0101{12πuln2u+18π3u12πvln2v18π3v}dudvu2v2=12π0101uln2uvln2vu2v2dudv+18π30101dudvu+v=12π{16π2+4ln212ζ(3)}+18π3×2ln2=π(14π2ln2112π2+2ln214ζ(3)) \begin{aligned} \int_0^\infty \left(\frac{\tan^{-1}x \ln x}{x}\right)^2\,dx & = \int_0^1 \int_0^1 \left(\int_0^\infty \frac{\ln^2x}{(1 + x^2u^2)(1 + x^2v^2)}\,dx\right)\,du\,dv \\ & = \int_0^1 \int_0^1 \left(\int_0^\infty \ln^2x\left\{ \frac{1}{x^2 + u^{-2}} - \frac{1}{x^2 + v^{-2}}\right\}\,dx \right)\,\frac{du\,dv}{u^2-v^2} \\ & = \int_0^1 \int_0^1 \left\{ \tfrac12\pi u\,\ln^2u + \tfrac18\pi^3u - \tfrac12\pi v\, \ln^2v - \tfrac18\pi^3v\right\} \frac{du\,dv}{u^2 - v^2} \\ & = \tfrac12\pi \int_0^1 \int_0^1 \frac{u \ln^2u - v\ln^2v}{u^2 - v^2}\,du\,dv + \tfrac18\pi^3 \int_0^1 \int_0^1 \frac{du\,dv}{u+v} \\ & = \tfrac12\pi\left\{ -\tfrac16\pi^2 + 4\ln2 - \tfrac12\zeta(3)\right\} + \tfrac18\pi^3 \times 2\ln2 \\ & = \pi\left(\tfrac14\pi^2\ln2 - \tfrac{1}{12}\pi^2 + 2\ln2 - \tfrac14\zeta(3)\right) \end{aligned} I initially relied on Mathematica to evaluate the double integral 0101uln2uvln2vu2v2dudv \int_0^1\int_0^1 \frac{u\ln^2u - v\ln^2v}{u^2 - v^2}\,du\,dv but here is a proper derivation.

0101uln2uvln2vu2v2dudv=201du0udvuln2uvln2vu2v2=201du0udv1u2v2vu(2lnw+ln2w)dw=0<v<w<u<12lnw+ln2wu(1u+v+1uv)=0<w<u<12lnw+ln2wuln(u+wuw)dwdu \begin{aligned} \int_0^1 \int_0^1 \frac{u\ln^2u - v\ln^2v}{u^2 - v^2}\,du\,dv & = 2\int_0^1 \,du \int_0^u\,dv \frac{u\ln^2u - v\ln^2v}{u^2-v^2} \\ & = 2\int_0^1\,du \int_0^u\,dv \frac{1}{u^2-v^2}\int_v^u (2\ln w + \ln^2w)\,dw \\ & = \int\int\int_{0 < v < w < u < 1}\frac{2\ln w + \ln^2w}{u}\left(\frac{1}{u+v} + \frac{1}{u-v}\right) \\ & = \iint_{0 < w < u < 1} \frac{2\ln w + \ln^2w}{u}\ln\left(\tfrac{u+w}{u-w}\right)\,dw\,du \end{aligned} Since it is easy to check (by differentiating back again) that (2lnw+ln2w)ln(u+wuw)dw  =  uln2wln(1+wu)+wln2wln(u+wuw)+uln2wln(1wu)+2ulnwLi2(wu)+2ulnwLi2(wu)2uLi3(wu)2uLi3(wu)+c \int (2\ln w + \ln^2w)\ln\left(\tfrac{u+w}{u-w}\right)\,dw \; = \; \begin{array}{l} \displaystyle u \ln^2w \ln\big(1 + \tfrac{w}{u}\big) + w\ln^2w \ln\big(\tfrac{u+w}{u-w}\big) \\ \displaystyle + u\ln^2w\ln\big(1 - \tfrac{w}{u}\big) + 2u\ln w \mathrm{Li}_2\big(-\tfrac{w}{u}\big) + 2u \ln w \mathrm{Li}_2\big(\tfrac{w}{u}\big) \\ \displaystyle - 2u\mathrm{Li}_3\big(-\tfrac{w}{u}\big) - 2u\mathrm{Li}_3\big(\tfrac{w}{u}\big) + c\end{array} it follows that 0u(2lnw+ln2w)ln(u+wuw)dw  =  16u(π2lnu+12ln2ln2u3ζ(3)) \int_0^u (2 \ln w + \ln^2w)\ln\left(\tfrac{u+w}{u-w}\right)\,dw \; = \; \tfrac16 u \left(\pi^2 \ln u + 12 \ln2 \ln^2u - 3 \zeta(3)\right) and so 0101uln2uvln2vu2v2dudv  =  1601(π2lnu+12ln2ln2u3ζ(3))du  =  16π2+4ln212ζ(3) \int_0^1 \int_0^1 \frac{u\ln^2u - v\ln^2v}{u^2 - v^2}\,du\,dv \; = \; \tfrac16\int_0^1\left(\pi^2 \ln u + 12 \ln2 \ln^2u - 3 \zeta(3)\right)\,du \; = \; -\tfrac16\pi^2 + 4\ln2 - \tfrac12\zeta(3) as required.

Mark Hennings - 4 years, 6 months ago

Problem 15 : Prove That

0cos(2nx)cosh2mxdx=π2Γ(m+in)2Γ(m)Γ(m+12) ; (m,n)R+ \int_{0}^{\infty} \dfrac{\cos (2nx)}{\cosh^{2m} x} \mathrm{d}x = \dfrac{\sqrt{\pi}}{2} \dfrac{|\Gamma(m+in)|^2}{\Gamma(m) \Gamma \left(m + \dfrac{1}{2} \right)} \ ; \ (m,n) \in \mathbb{R}^+

This problem has been solved by Mark Hennings.

Ishan Singh - 4 years, 5 months ago

Log in to reply

Note that B(z,w)=0tz1(1+t)z+wdt  =  (01+1)tz1(1+t)z+wdt=01tz1(1+t)z+wdt+10t1z(1+t1)z+w(t2dt)=01tz1+tw1(1+t)z+wdt \begin{aligned} B(z,w) & = \int_0^\infty \frac{t^{z-1}}{(1+t)^{z+w}}\,dt \; = \; \left(\int_0^1 + \int_1^\infty\right) \frac{t^{z-1}}{(1+t)^{z+w}}\,dt \\ & = \int_0^1 \frac{t^{z-1}}{(1+t)^{z+w}}\,dt + \int_1^0 \frac{t^{1-z}}{(1+t^{-1})^{z+w}} \big(-t^{-2}\,dt\big) \\ & = \int_0^1 \frac{t^{z-1} + t^{w-1}}{(1+t)^{z+w}}\,dt \end{aligned} for all Rez,Rew>0\mathfrak{Re}\,z\,,\,\mathfrak{Re}\,w \,>\, 0, so that (with the substitution t=e2xt = e^{-2x}) B(m+in,min)=01tm+in1+tmin1(1+t)2mdt  =  01tm1(tin+tin)(1+t)2mdt=0e2(1m)x(e2inx+e2inx)(1+e2x)2m(2e2x)dx=40e2mxcos2nx(1+e2x)2mdx  =  40cos2nx(ex+ex)2mdx=41m0cos2nxcosh2mxdx \begin{aligned} B(m+in,m-in) & = \int_0^1 \frac{t^{m+in-1} + t^{m-in-1}}{(1+t)^{2m}}\,dt \; = \; \int_0^1 \frac{t^{m-1}(t^{in} + t^{-in})}{(1 + t)^{2m}}\,dt \\ & = \int_\infty^0 \frac{e^{2(1-m)x} \big(e^{-2inx} + e^{2inx}\big)}{(1 + e^{-2x})^{2m}}\,(-2e^{-2x})\,dx \\ & = 4\int_0^\infty \frac{e^{-2mx} \cos2nx}{(1 + e^{-2x})^{2m}}\,dx \; = \; 4\int_0^\infty \frac{\cos 2nx}{(e^x + e^{-x})^{2m}}\,dx \\ & = 4^{1-m}\int_0^\infty \frac{\cos 2nx}{\cosh^{2m}x}\,dx \end{aligned} for any real nn and m>0m > 0, so that 0cos2nxcosh2mxdx=4m1B(m+in,min)  =  4m1Γ(2m)Γ(m+in)Γ(min)=π2Γ(m)Γ(m+12)Γ(m+in)2 \begin{aligned} \int_0^\infty \frac{\cos 2nx}{\cosh^{2m}x}\,dx & = 4^{m-1} B(m+in,m-in) \; = \; \frac{4^{m-1}}{\Gamma(2m)} \Gamma(m+in) \Gamma(m-in) \\ & = \frac{\sqrt{\pi}}{2\Gamma(m)\Gamma(m+\frac12)} \big|\Gamma(m+in)\big|^2 \end{aligned} for any real nn and any m>0m > 0, using the duplication formula Γ(2m)  =  12π22m12Γ(m)Γ(m+12) \Gamma(2m) \;= \; \tfrac{1}{\sqrt{2\pi}} 2^{2m - \frac12} \Gamma(m) \Gamma(m+\tfrac12)

Mark Hennings - 4 years, 5 months ago

Problem 17 :

Find a closed form for 0sin2n+1xxdx\displaystyle \int_0^\infty \frac{\sin^{2n+1}x}{x}\, dx for n0\, n\ge 0

This problem has been solved by Ishan SIngh

Aditya Narayan Sharma - 4 years, 5 months ago

Log in to reply

Since,

sinx=eixeix2i \sin x = \dfrac{e^{ix} - e^{-ix}}{2i}

We have,

sin2n+1x=(1)n22nr=0n(1)r(2n+1r)sin(2r+1)x \sin^{2n+1} x = \dfrac{(-1)^n}{2^{2n}} \sum_{r=0}^{n} (-1)^r \dbinom{2n+1}{r} \sin (2r+1)x

    0sin2n+1xx dx=(1)n22nr=0n(1)r(2n+1r)0sin(2r+1)xx dx\implies \int_{0}^{\infty} \dfrac{\sin^{2n+1} x}{x} \ \mathrm{d}x = \dfrac{(-1)^n}{2^{2n}} \sum_{r=0}^{n} (-1)^r \dbinom{2n+1}{r} \int_{0}^{\infty} \dfrac{\sin (2r+1)x}{x} \ \mathrm{d}x

=(1)nπ22n+1r=0n(1)r(2n+1r) = \dfrac{(-1)^n \pi}{2^{2n+1}} \sum_{r=0}^{n} (-1)^r \dbinom{2n+1}{r}

=(1)nπ22n+1r=0n((1)r(2nr)(1)r1(2nr1)) = \dfrac{(-1)^n \pi}{2^{2n+1}} \sum_{r=0}^{n} \left( (-1)^r \dbinom{2n}{r} - (-1)^{r-1} \dbinom{2n}{r-1} \right)

Clearly, the above sum telescopes. Evaluating it, we have,

0sin2n+1xx dx=(1)nπ22n+1(1)n(2nn) \int_{0}^{\infty} \dfrac{\sin^{2n+1} x}{x} \ \mathrm{d}x = \dfrac{(-1)^n \pi}{2^{2n+1}} (-1)^n \dbinom{2n}{n}

0sin2n+1xx dx=π22n+1(2nn)  nZ+ \therefore \int_{0}^{\infty} \dfrac{\sin^{2n+1} x}{x} \ \mathrm{d}x = \dfrac{\pi}{2^{2n+1}} \dbinom{2n}{n} \ \forall \ n \in \mathbb{Z}^+

Ishan Singh - 4 years, 5 months ago

Log in to reply

As an alternative,

I=0sin2n+1xxdx=00exysin2n+1xdxdy\displaystyle I=\int_0^\infty \frac{\sin^{2n+1}x}{x}\,dx = \int_0^\infty\int_0^\infty e^{-xy}\sin^{2n+1}x\,dx\,dy

Now by IBP two times we can create a recurrence relation and thus evaluate,

Jn=0eaxsin2n+1xdx=(2n+1)!r=1n(a2+(2r+1)2)\displaystyle J_n = \int_0^\infty e^{-ax}\sin^{2n+1}x\,dx = \frac{(2n+1)!}{\prod_{r=1}^{n}(a^2 +(2r+1)^2)}

Thus, I=(2n+1)!0dyr=1n(a2+(2r+1)2)\displaystyle I=(2n+1)!\int_0^\infty \frac{dy}{\prod_{r=1}^{n}(a^2 +(2r+1)^2)}

The expression 1r=1n(a2+(2r+1)2)=r=0nAry2+(2r+1)2\displaystyle \frac{1}{\prod_{r=1}^{n}(a^2 +(2r+1)^2)} = \sum_{r=0}^n \frac{A_r}{y^2+(2r+1)^2}

where the coefficients can be determined by the cover-up rule as Ak=(1)k22n2k+1(nk)!(n+k+1)!\displaystyle A_k = \frac{(-1)^k}{2^{2n}}\frac{2k+1}{(n-k)!(n+k+1)!}

So, I=(2n+1)!k=0n0Aky2+(2k+1)2dy=π22n+1k=0n(1)k(2n+1nk)=π22n+1(2nn)\displaystyle I=(2n+1)!\sum_{k=0}^n \int_0^\infty \frac{A_k}{y^2+(2k+1)^2}\,dy = \frac{\pi}{2^{2n+1}} \sum_{k=0}^{n} (-1)^k \binom{2n+1}{n-k} = \frac{\pi}{2^{2n+1}}\binom{2n}{n}

Aditya Narayan Sharma - 4 years, 5 months ago

Log in to reply

@Aditya Narayan Sharma That technique looks familiar!

Mark Hennings - 4 years, 5 months ago

It is interesting to note that the sum r=0m(1)r(nr)  =  (1)m(n1m)0m<n \sum_{r=0}^m (-1)^r {n \choose r} \; = \; (-1)^m{n-1 \choose m} \hspace{2cm} 0 \le m < n can be proved by considering the coefficients of xmx^m on both sides of the identity (1+x)n1  =  (1+x)1×(1+x)nx<1  . (1 + x)^{n-1} \; = \; (1 + x)^{-1} \times (1+x)^n \hspace{1cm} |x| < 1 \;.

Mark Hennings - 4 years, 5 months ago

(+1) Nice ! I have another method so all total we could have atleast 2 real methods to evaluate this.

Aditya Narayan Sharma - 4 years, 5 months ago

Problem 22:

Evaluate 014πln(1+tanx)dx \int_0^{\frac14\pi} \ln(1 + \tan x)\,dx

This problem has been solved by Rohinth M.Athreya.

Mark Hennings - 4 years, 5 months ago

Log in to reply

0π4ln(1+tan(π4x)dx \displaystyle \large \int_{0}^{\frac{\pi}{4}} ln(1+\tan(\frac{\pi}{4} - x)\, dx

0π4ln(21+tanxdx \displaystyle \large \int_{0}^{\frac{\pi}{4}} ln(\frac{2}{1+\tan x}\,dx

if we denote the integral by II. it is also πln24I\frac{\pi ln2}{4} -I

and so the integral is πln28\frac{\pi ln2}{8} .

Rohith M.Athreya - 4 years, 5 months ago

Problem 23:

Evaluate the indefinite integral, x((1+x2)1012(2+x2)3012)12012dx\displaystyle \large \int \frac{x}{((1+x^{2})^{1012} (2+x^2)^{3012})^{\frac{1}{2012}}} \, dx.

This problem has been solved by Mark Hennings.

Rohith M.Athreya - 4 years, 5 months ago

Log in to reply

Start with the substitution y=x2+1y = x^2 + 1. Then Ip(x)=xdx(1+x2)1p(2+x2)1+p  =  12dyy1p(1+y)1+p=12(y1+y)p1dy(1+y)2\begin{aligned} I_{p}(x) & = \int \frac{x\,dx}{(1 + x^2)^{1-p}(2 + x^2)^{1 + p}} \; = \; \tfrac12\int \frac{dy}{y^{1-p}(1 + y)^{1+p}} \\ & = \tfrac12 \int \left(\frac{y}{1+y}\right)^{p-1}\,\frac{dy}{(1+y)^2} \end{aligned} Then the substitution z=y1+yz = \frac{y}{1+y} gives Ip(x)  =  12zp1dz  =  12pzp+c  =  12p(x2+1x2+2)p+c I_p(x) \; = \; \tfrac12\int z^{p-1}\,dz \; = \; \tfrac{1}{2p}z^p + c \; = \; \tfrac{1}{2p}\left(\tfrac{x^2+1}{x^2+2}\right)^p + c In this case, we have p=10002012=250503p = \tfrac{1000}{2012} = \tfrac{250}{503}, and hence I250503(x)  =  503500(x2+1x2+2)250503+c I_{\frac{250}{503}}(x) \; = \; \tfrac{503}{500} \left(\tfrac{x^2+1}{x^2+2}\right)^{\frac{250}{503}} + c

Mark Hennings - 4 years, 5 months ago

Problem 21:

Evaluate 0π2ln2sinxln2cosxdx\displaystyle\int_0^\frac{\pi}{2}\ln^2{\sin{x}}\ln^2{\cos{x}}\,dx

This problem has been solved by Mark Hennings.

First Last - 4 years, 5 months ago

Log in to reply

Start with f(a,b)  =  012πsina1xcosb1xdx  =  12B(12a,12b)  =  Γ(12a)Γ(12b)2Γ(12a+12b) f(a,b) \; = \; \int_0^{\frac12\pi} \sin^{a-1}x \cos^{b-1}x\,dx \; = \; \tfrac12B\big(\tfrac12a,\tfrac12b\big) \; = \; \frac{\Gamma(\frac12a)\Gamma(\frac12b)}{2\Gamma(\frac12a+\frac12b)} for a,b>0a,b > 0. Then fa=12f(a,b)[ψ(12a)ψ(12a+12b)]2fa2=14f(a,b)[ψ(12a)ψ(12a+12b)+[ψ(12a)ψ(12a+12b)]2]\begin{aligned} \frac{\partial f}{\partial a} & = \tfrac12 f(a,b)\Big[\psi\big(\tfrac12a\big) - \psi\big(\tfrac12a+\tfrac12b\big)\Big] \\ \frac{\partial^2 f}{\partial a^2} & = \tfrac14 f(a,b) \Big[ \psi'\big(\tfrac12a\big) - \psi'\big(\tfrac12a+\tfrac12b\big) + \big[\psi\big(\tfrac12a\big) - \psi\big(\tfrac12a+\tfrac12b\big)\big]^2\Big] \end{aligned} and so g(b)  =  2fa2(1,b)  =  18A(b)B(b) g(b) \; = \; \frac{\partial^2 f}{\partial a^2}(1,b) \; = \; \tfrac18 A(b)B(b) where A(b)  =  Γ(12)Γ(12b)Γ(12+12b)  =  πΓ(12b)Γ(12+12b)B(b)  =  ψ(12)ψ(12+12b)+[ψ(12)ψ(12+12b)]2 A(b) \; = \; \frac{\Gamma(\frac12)\Gamma(\frac12b)}{\Gamma(\frac12+\frac12b)} \; = \; \frac{\sqrt{\pi}\Gamma(\frac12b)}{\Gamma(\frac12+\frac12b)} \hspace{1cm} B(b) \; = \; \psi'\big(\tfrac12\big) - \psi'\big(\tfrac12+\tfrac12b\big) + \big[\psi\big(\tfrac12\big) - \psi\big(\tfrac12+\tfrac12b\big)\big]^2 Thus A(b)=12A(b)[ψ(12b)ψ(12+12b)]A(b)=14A(b)[ψ(12b)ψ(12+12b)+[ψ(12b)ψ(12+12b)]2]\begin{aligned} A'(b) & = \tfrac12A(b)\big[\psi\big(\tfrac12b\big) - \psi\big(\tfrac12+\tfrac12b\big)\big] \\ A''(b) & = \tfrac14A(b) \Big[ \psi'\big(\tfrac12b\big) - \psi'\big(\tfrac12 + \tfrac12b\big) + \big[\psi\big(\tfrac12b\big) - \psi\big(\tfrac12+\tfrac12b\big)\big]^2\Big] \end{aligned} and hence A(1)  =  πA(1)  =  πln2A(1)  =  112π3+π(ln2)2 A(1) \; = \; \pi \hspace{1.5cm} A'(1) \; = \; -\pi \ln2 \hspace{1.5cm} A''(1) \; = \; \tfrac{1}{12}\pi^3 + \pi(\ln2)^2 Also B(b)=12ψ(12+12b)[ψ(12)ψ(12+12b)]ψ(12+12b)B(b)=14ψ(12+12b)+12ψ(12+12b)212[ψ(12)ψ(12+12b)]ψ(12+12b)\begin{aligned} B'(b) & = -\tfrac12\psi''\big(\tfrac12+\tfrac12b\big) - \big[\psi\big(\tfrac12\big) - \psi\big(\tfrac12+\tfrac12b\big)\big]\psi'\big(\tfrac12+\tfrac12b\big) \\ B''(b) & = -\tfrac14\psi'''\big(\tfrac12+\tfrac12b\big) + \tfrac12\psi'\big(\tfrac12+\tfrac12b\big)^2 - \tfrac12\big[\psi\big(\tfrac12\big) - \psi\big(\tfrac12+\tfrac12b\big)\big]\psi''\big(\tfrac12+\tfrac12b\big) \end{aligned} and hence B(1)  =  13π2+4(ln2)2B(1)  =  13π2ln2+ζ(3)B(1)  =  1360π42ζ(3)ln2 B(1) \; =\; \tfrac13\pi^2 + 4(\ln2)^2 \hspace{1.5cm} B'(1) \; = \; \tfrac13\pi^2\ln2 + \zeta(3) \hspace{1.5cm} B''(1) \; = \; -\tfrac{1}{360}\pi^4 - 2 \zeta(3)\ln2 and hence 012πln2(sinx)ln2(cosx)dx=4fa2b2a=b=1  =  g(1)=18[A(1)B(1)+2A(1)B(1)+A(1)B(1)]=1320π5+12π(ln2)412πζ(3)ln2\begin{aligned} \int_0^{\frac12\pi} \ln^2(\sin x) \ln^2(\cos x)\,dx & = \frac{\partial^4 f}{\partial a^2 \partial b^2} \Big|_{a=b=1} \; = \; g''(1) \\ & = \tfrac18\big[A''(1)B(1) + 2A'(1)B'(1) + A(1)B''(1)\big] \\ & = \tfrac{1}{320}\pi^5 + \tfrac12\pi(\ln2)^4 - \tfrac12\pi\zeta(3)\ln2 \end{aligned}

Mark Hennings - 4 years, 5 months ago

PROBLEM 24:

Show that 0eax2bx2dx  =  12πae2ab \int_0^\infty e^{-ax^2 - bx^{-2}}\,dx \; = \; \tfrac12\sqrt{\tfrac{\pi}{a}} e^{-2\sqrt{ab}} for any a,b>0a,b > 0.

Mark Hennings - 4 years, 5 months ago

Log in to reply

Let,

I=0eax2bx2 dx\text{I} = \int_{0}^{\infty} e^{-ax^2 - \frac{b}{x^{2}}} \ \mathrm{d}x

=12eax2bx2 dx = \dfrac{1}{2} \int_{-\infty}^{\infty} e^{-ax^2 - \frac{b}{x^{2}}} \ \mathrm{d}x

Sunstituting xxax \mapsto \dfrac{x}{\sqrt{a}}, we have,

I=12aex2abx2 dx \text{I}= \dfrac{1}{2\sqrt{a}} \int_{-\infty}^{\infty} e^{-x^2 - \frac{ab}{x^{2}}} \ \mathrm{d}x

=e2ab2ae(xabx)2 dx = \dfrac{e^{-2\sqrt{ab}}}{2\sqrt{a}} \int_{-\infty}^{\infty} e^{- \left(x - \frac{\sqrt{ab}}{x} \right)^2 } \ \mathrm{d}x

Using Glasser's Master Theorem, we have,

I=e2ab2aex2 dx \text{I} = \dfrac{e^{-2\sqrt{ab}}}{2\sqrt{a}} \int_{-\infty}^{\infty} e^{-x^2 } \ \mathrm{d}x

=12πae2ab = \dfrac{1}{2} \sqrt{\dfrac{\pi}{a}} e^{-2\sqrt{ab}} \quad \square

Ishan Singh - 4 years, 5 months ago

Log in to reply

You don't need a master theorem!

Completing the square 0eax2bx2dx  =  e2ab0e(axbx1)2dx \int_0^\infty e^{-a x^2 - b x^{-2}}\,dx \; = \; e^{-2\sqrt{ab}}\int_0^\infty e^{-(\sqrt{a} x - \sqrt{b} x^{-1})^2}\,dx Now the substitution y=bax1y \,=\, \sqrt{\tfrac{b}{a}} x^{-1} yields 0e(axbx1)2dx=0e(byay)2×(bay2)dy=ba0x2e(axbx1)2dy=120(1+bax2)e(axbx1)2dx=12a0(a+bx2)e(axbx1)2dx\begin{aligned} \int_0^\infty e^{-(\sqrt{a}x - \sqrt{b} x^{-1})^2}\,dx & = \int_\infty^0 e^{-(\sqrt{b}y - \sqrt{a}y)^2} \times \big(-\sqrt{\tfrac{b}{a}}y^{-2}\big)\,dy \\ & = \sqrt{\tfrac{b}{a}}\int_0^\infty x^{-2} e^{-(\sqrt{a}x - \sqrt{b}x^{-1})^2}\,dy \\ & = \frac12\int_0^\infty \left(1 + \sqrt{\tfrac{b}{a}}x^{-2}\right) e^{-(\sqrt{a}x - \sqrt{b}x^{-1})^2}\,dx \\ & = \frac{1}{2\sqrt{a}}\int_0^\infty \left(\sqrt{a} + \sqrt{b}x^{-2}\right) e^{-(\sqrt{a} x -\sqrt{b}x^{-1})^2}\,dx \end{aligned} The further substitution u=axbx1u = \sqrt{a}x - \sqrt{b}x^{-1} gives 0e(axbx1)2dx  =  12aReu2du  =  12πa \int_0^\infty e^{-(\sqrt{a}x - \sqrt{b} x^{-1})^2}\,dx \; = \; \frac{1}{2\sqrt{a}}\int_{\mathbb{R}} e^{-u^2}\,du \; = \; \tfrac12\sqrt{\tfrac{\pi}{a}} which implies that 0eax2bx2dx  =  12πae2ab \int_0^\infty e^{-a x^2 - b x^{-2}}\,dx \; = \; \tfrac12\sqrt{\tfrac{\pi}{a}}e^{-2\sqrt{ab}}

Mark Hennings - 4 years, 5 months ago

Log in to reply

@Mark Hennings I know. I have solved it that way also, but I thought solving it this way gives it a new perspective and also introduces to a new Theorem.

Ishan Singh - 4 years, 5 months ago

Solve LaTex: \displaysize \large \int _((cos \phi)^-1)^((cos \phi)^-1)\((e^(\delta*x))/(\sqrt (1-(x^2)))dx

×

Problem Loading...

Note Loading...

Set Loading...